Top Banner
. . . . . . . . . . . . . . . . . . . . . . . . . . . . . . . . . . . . . . . . . . . . . . . . . . . . . . . . . . . . . . . . . . . . . . . . . . . . . . . . . . . . . . . . . . Recap . . . . . Bayesian Tests . . . . . Bayesian Intervals . . . P1 . . . . . . . . P2 . . . . P3 . . . . P4 . . Biostatistics 602 - Statistical Inference Lecture 25 Bayesian Test & Practice Problems Hyun Min Kang April 18th, 2013 Hyun Min Kang Biostatistics 602 - Lecture 25 April 18th, 2013 1 / 34
134

Biostatistics 602 - Statistical Inference Lecture 25 ... · Bayesian Tests • Hypothesis testing problems can be formulated in a Bayesian model • Bayesian model includes • Sampling

Oct 03, 2020

Download

Documents

dariahiddleston
Welcome message from author
This document is posted to help you gain knowledge. Please leave a comment to let me know what you think about it! Share it to your friends and learn new things together.
Transcript
Page 1: Biostatistics 602 - Statistical Inference Lecture 25 ... · Bayesian Tests • Hypothesis testing problems can be formulated in a Bayesian model • Bayesian model includes • Sampling

..........

.....

......

.....

.....

.....

......

.....

.....

.....

......

.....

.....

.....

......

.....

......

.....

.....

.

. . . .Recap

. . . . .Bayesian Tests

. . . . .Bayesian Intervals

. . .P1

. . . . . . . .P2

. . . .P3

. . . .P4

.

......

Biostatistics 602 - Statistical InferenceLecture 25

Bayesian Test & Practice Problems

Hyun Min Kang

April 18th, 2013

Hyun Min Kang Biostatistics 602 - Lecture 25 April 18th, 2013 1 / 34

Page 2: Biostatistics 602 - Statistical Inference Lecture 25 ... · Bayesian Tests • Hypothesis testing problems can be formulated in a Bayesian model • Bayesian model includes • Sampling

..........

.....

......

.....

.....

.....

......

.....

.....

.....

......

.....

.....

.....

......

.....

......

.....

.....

.

. . . .Recap

. . . . .Bayesian Tests

. . . . .Bayesian Intervals

. . .P1

. . . . . . . .P2

. . . .P3

. . . .P4

Last Lecture

• What is an E-M algorithm?• When would the E-M algorithm be useful?• Is MLE via E-M algorithm always guaranteed to converge?• What are the practical limitations of the E-M algorithm?

Hyun Min Kang Biostatistics 602 - Lecture 25 April 18th, 2013 2 / 34

Page 3: Biostatistics 602 - Statistical Inference Lecture 25 ... · Bayesian Tests • Hypothesis testing problems can be formulated in a Bayesian model • Bayesian model includes • Sampling

..........

.....

......

.....

.....

.....

......

.....

.....

.....

......

.....

.....

.....

......

.....

......

.....

.....

.

. . . .Recap

. . . . .Bayesian Tests

. . . . .Bayesian Intervals

. . .P1

. . . . . . . .P2

. . . .P3

. . . .P4

Last Lecture

• What is an E-M algorithm?

• When would the E-M algorithm be useful?• Is MLE via E-M algorithm always guaranteed to converge?• What are the practical limitations of the E-M algorithm?

Hyun Min Kang Biostatistics 602 - Lecture 25 April 18th, 2013 2 / 34

Page 4: Biostatistics 602 - Statistical Inference Lecture 25 ... · Bayesian Tests • Hypothesis testing problems can be formulated in a Bayesian model • Bayesian model includes • Sampling

..........

.....

......

.....

.....

.....

......

.....

.....

.....

......

.....

.....

.....

......

.....

......

.....

.....

.

. . . .Recap

. . . . .Bayesian Tests

. . . . .Bayesian Intervals

. . .P1

. . . . . . . .P2

. . . .P3

. . . .P4

Last Lecture

• What is an E-M algorithm?• When would the E-M algorithm be useful?

• Is MLE via E-M algorithm always guaranteed to converge?• What are the practical limitations of the E-M algorithm?

Hyun Min Kang Biostatistics 602 - Lecture 25 April 18th, 2013 2 / 34

Page 5: Biostatistics 602 - Statistical Inference Lecture 25 ... · Bayesian Tests • Hypothesis testing problems can be formulated in a Bayesian model • Bayesian model includes • Sampling

..........

.....

......

.....

.....

.....

......

.....

.....

.....

......

.....

.....

.....

......

.....

......

.....

.....

.

. . . .Recap

. . . . .Bayesian Tests

. . . . .Bayesian Intervals

. . .P1

. . . . . . . .P2

. . . .P3

. . . .P4

Last Lecture

• What is an E-M algorithm?• When would the E-M algorithm be useful?• Is MLE via E-M algorithm always guaranteed to converge?

• What are the practical limitations of the E-M algorithm?

Hyun Min Kang Biostatistics 602 - Lecture 25 April 18th, 2013 2 / 34

Page 6: Biostatistics 602 - Statistical Inference Lecture 25 ... · Bayesian Tests • Hypothesis testing problems can be formulated in a Bayesian model • Bayesian model includes • Sampling

..........

.....

......

.....

.....

.....

......

.....

.....

.....

......

.....

.....

.....

......

.....

......

.....

.....

.

. . . .Recap

. . . . .Bayesian Tests

. . . . .Bayesian Intervals

. . .P1

. . . . . . . .P2

. . . .P3

. . . .P4

Last Lecture

• What is an E-M algorithm?• When would the E-M algorithm be useful?• Is MLE via E-M algorithm always guaranteed to converge?• What are the practical limitations of the E-M algorithm?

Hyun Min Kang Biostatistics 602 - Lecture 25 April 18th, 2013 2 / 34

Page 7: Biostatistics 602 - Statistical Inference Lecture 25 ... · Bayesian Tests • Hypothesis testing problems can be formulated in a Bayesian model • Bayesian model includes • Sampling

..........

.....

......

.....

.....

.....

......

.....

.....

.....

......

.....

.....

.....

......

.....

......

.....

.....

.

. . . .Recap

. . . . .Bayesian Tests

. . . . .Bayesian Intervals

. . .P1

. . . . . . . .P2

. . . .P3

. . . .P4

Overview of E-M Algorithm (cont’d)

.Objective..

......

• Maximize L(θ|y) or l(θ|y).• Let f(y, z|θ) denotes the pdf of complete data. In E-M algorithm,

rather than working with l(θ|y) directly, we work with the surrogatefunction

Q(θ|θ(r)) = E[log f(y,Z|θ)|y, θ(r)

]where θ(r) is the estimation of θ in r-th iteration.

• Q(θ|θ(r)) is the expected log-likelihood of complete data, conditioningon the observed data and θ(r).

Hyun Min Kang Biostatistics 602 - Lecture 25 April 18th, 2013 3 / 34

Page 8: Biostatistics 602 - Statistical Inference Lecture 25 ... · Bayesian Tests • Hypothesis testing problems can be formulated in a Bayesian model • Bayesian model includes • Sampling

..........

.....

......

.....

.....

.....

......

.....

.....

.....

......

.....

.....

.....

......

.....

......

.....

.....

.

. . . .Recap

. . . . .Bayesian Tests

. . . . .Bayesian Intervals

. . .P1

. . . . . . . .P2

. . . .P3

. . . .P4

Key Steps of E-M algorithm.Expectation Step..

......

• Compute Q(θ|θ(r)).• This typically involves in estimating the conditional distribution Z|Y,

assuming θ = θ(r).• After computing Q(θ|θ(r)), move to the M-step

.Maximization Step..

......

• Maximize Q(θ|θ(r)) with respect to θ.• The arg maxθ Q(θ|θ(r)) will be the (r + 1)-th θ to be fed into the

E-step.• Repeat E-step until convergence

Hyun Min Kang Biostatistics 602 - Lecture 25 April 18th, 2013 4 / 34

Page 9: Biostatistics 602 - Statistical Inference Lecture 25 ... · Bayesian Tests • Hypothesis testing problems can be formulated in a Bayesian model • Bayesian model includes • Sampling

..........

.....

......

.....

.....

.....

......

.....

.....

.....

......

.....

.....

.....

......

.....

......

.....

.....

.

. . . .Recap

. . . . .Bayesian Tests

. . . . .Bayesian Intervals

. . .P1

. . . . . . . .P2

. . . .P3

. . . .P4

Does E-M iteration converge to MLE?

.Theorem 7.2.20 - Monotonic EM sequence..

......

The sequence θ(r) defined by the E-M procedure satisfiesL(θ(r+1)|y

)≥ L

(θ(r)|y

)with equality holding if and only if successive iterations yield the samevalue of the maximized expected complete-data log likelihood, that is

E[log L

(θ(r+1)|y,Z

)|θ(r), y

]= E

[log L

(θ(r)|y,Z

)|θ(r), y

]Theorem 7.5.2 further guarantees that L(θ(r)|y) converges monotonicallyto L(θ|y) for some stationary point θ.

Hyun Min Kang Biostatistics 602 - Lecture 25 April 18th, 2013 5 / 34

Page 10: Biostatistics 602 - Statistical Inference Lecture 25 ... · Bayesian Tests • Hypothesis testing problems can be formulated in a Bayesian model • Bayesian model includes • Sampling

..........

.....

......

.....

.....

.....

......

.....

.....

.....

......

.....

.....

.....

......

.....

......

.....

.....

.

. . . .Recap

. . . . .Bayesian Tests

. . . . .Bayesian Intervals

. . .P1

. . . . . . . .P2

. . . .P3

. . . .P4

Bayesian Tests

• Hypothesis testing problems can be formulated in a Bayesian model

• Bayesian model includes• Sampling distribution f(x|θ)• Prior distribution π(θ)

• Bayesian hypothesis testing is based on the posterior probability• In Frequentist’s framework, posterior probability cannot be calculated.• In Bayesian framework, the probability of H0 and H1 can be calculated

• Pr(θ ∈ Ω0|x) = Pr(H0 is true)• Pr(θ ∈ Ωc

0|x) = Pr(H1 is true)• Rejection region can be determined directly based on the posterior

probability

Hyun Min Kang Biostatistics 602 - Lecture 25 April 18th, 2013 6 / 34

Page 11: Biostatistics 602 - Statistical Inference Lecture 25 ... · Bayesian Tests • Hypothesis testing problems can be formulated in a Bayesian model • Bayesian model includes • Sampling

..........

.....

......

.....

.....

.....

......

.....

.....

.....

......

.....

.....

.....

......

.....

......

.....

.....

.

. . . .Recap

. . . . .Bayesian Tests

. . . . .Bayesian Intervals

. . .P1

. . . . . . . .P2

. . . .P3

. . . .P4

Bayesian Tests

• Hypothesis testing problems can be formulated in a Bayesian model• Bayesian model includes

• Sampling distribution f(x|θ)• Prior distribution π(θ)

• Bayesian hypothesis testing is based on the posterior probability• In Frequentist’s framework, posterior probability cannot be calculated.• In Bayesian framework, the probability of H0 and H1 can be calculated

• Pr(θ ∈ Ω0|x) = Pr(H0 is true)• Pr(θ ∈ Ωc

0|x) = Pr(H1 is true)• Rejection region can be determined directly based on the posterior

probability

Hyun Min Kang Biostatistics 602 - Lecture 25 April 18th, 2013 6 / 34

Page 12: Biostatistics 602 - Statistical Inference Lecture 25 ... · Bayesian Tests • Hypothesis testing problems can be formulated in a Bayesian model • Bayesian model includes • Sampling

..........

.....

......

.....

.....

.....

......

.....

.....

.....

......

.....

.....

.....

......

.....

......

.....

.....

.

. . . .Recap

. . . . .Bayesian Tests

. . . . .Bayesian Intervals

. . .P1

. . . . . . . .P2

. . . .P3

. . . .P4

Bayesian Tests

• Hypothesis testing problems can be formulated in a Bayesian model• Bayesian model includes

• Sampling distribution f(x|θ)

• Prior distribution π(θ)

• Bayesian hypothesis testing is based on the posterior probability• In Frequentist’s framework, posterior probability cannot be calculated.• In Bayesian framework, the probability of H0 and H1 can be calculated

• Pr(θ ∈ Ω0|x) = Pr(H0 is true)• Pr(θ ∈ Ωc

0|x) = Pr(H1 is true)• Rejection region can be determined directly based on the posterior

probability

Hyun Min Kang Biostatistics 602 - Lecture 25 April 18th, 2013 6 / 34

Page 13: Biostatistics 602 - Statistical Inference Lecture 25 ... · Bayesian Tests • Hypothesis testing problems can be formulated in a Bayesian model • Bayesian model includes • Sampling

..........

.....

......

.....

.....

.....

......

.....

.....

.....

......

.....

.....

.....

......

.....

......

.....

.....

.

. . . .Recap

. . . . .Bayesian Tests

. . . . .Bayesian Intervals

. . .P1

. . . . . . . .P2

. . . .P3

. . . .P4

Bayesian Tests

• Hypothesis testing problems can be formulated in a Bayesian model• Bayesian model includes

• Sampling distribution f(x|θ)• Prior distribution π(θ)

• Bayesian hypothesis testing is based on the posterior probability• In Frequentist’s framework, posterior probability cannot be calculated.• In Bayesian framework, the probability of H0 and H1 can be calculated

• Pr(θ ∈ Ω0|x) = Pr(H0 is true)• Pr(θ ∈ Ωc

0|x) = Pr(H1 is true)• Rejection region can be determined directly based on the posterior

probability

Hyun Min Kang Biostatistics 602 - Lecture 25 April 18th, 2013 6 / 34

Page 14: Biostatistics 602 - Statistical Inference Lecture 25 ... · Bayesian Tests • Hypothesis testing problems can be formulated in a Bayesian model • Bayesian model includes • Sampling

..........

.....

......

.....

.....

.....

......

.....

.....

.....

......

.....

.....

.....

......

.....

......

.....

.....

.

. . . .Recap

. . . . .Bayesian Tests

. . . . .Bayesian Intervals

. . .P1

. . . . . . . .P2

. . . .P3

. . . .P4

Bayesian Tests

• Hypothesis testing problems can be formulated in a Bayesian model• Bayesian model includes

• Sampling distribution f(x|θ)• Prior distribution π(θ)

• Bayesian hypothesis testing is based on the posterior probability

• In Frequentist’s framework, posterior probability cannot be calculated.• In Bayesian framework, the probability of H0 and H1 can be calculated

• Pr(θ ∈ Ω0|x) = Pr(H0 is true)• Pr(θ ∈ Ωc

0|x) = Pr(H1 is true)• Rejection region can be determined directly based on the posterior

probability

Hyun Min Kang Biostatistics 602 - Lecture 25 April 18th, 2013 6 / 34

Page 15: Biostatistics 602 - Statistical Inference Lecture 25 ... · Bayesian Tests • Hypothesis testing problems can be formulated in a Bayesian model • Bayesian model includes • Sampling

..........

.....

......

.....

.....

.....

......

.....

.....

.....

......

.....

.....

.....

......

.....

......

.....

.....

.

. . . .Recap

. . . . .Bayesian Tests

. . . . .Bayesian Intervals

. . .P1

. . . . . . . .P2

. . . .P3

. . . .P4

Bayesian Tests

• Hypothesis testing problems can be formulated in a Bayesian model• Bayesian model includes

• Sampling distribution f(x|θ)• Prior distribution π(θ)

• Bayesian hypothesis testing is based on the posterior probability• In Frequentist’s framework, posterior probability cannot be calculated.

• In Bayesian framework, the probability of H0 and H1 can be calculated• Pr(θ ∈ Ω0|x) = Pr(H0 is true)• Pr(θ ∈ Ωc

0|x) = Pr(H1 is true)• Rejection region can be determined directly based on the posterior

probability

Hyun Min Kang Biostatistics 602 - Lecture 25 April 18th, 2013 6 / 34

Page 16: Biostatistics 602 - Statistical Inference Lecture 25 ... · Bayesian Tests • Hypothesis testing problems can be formulated in a Bayesian model • Bayesian model includes • Sampling

..........

.....

......

.....

.....

.....

......

.....

.....

.....

......

.....

.....

.....

......

.....

......

.....

.....

.

. . . .Recap

. . . . .Bayesian Tests

. . . . .Bayesian Intervals

. . .P1

. . . . . . . .P2

. . . .P3

. . . .P4

Bayesian Tests

• Hypothesis testing problems can be formulated in a Bayesian model• Bayesian model includes

• Sampling distribution f(x|θ)• Prior distribution π(θ)

• Bayesian hypothesis testing is based on the posterior probability• In Frequentist’s framework, posterior probability cannot be calculated.• In Bayesian framework, the probability of H0 and H1 can be calculated

• Pr(θ ∈ Ω0|x) = Pr(H0 is true)• Pr(θ ∈ Ωc

0|x) = Pr(H1 is true)• Rejection region can be determined directly based on the posterior

probability

Hyun Min Kang Biostatistics 602 - Lecture 25 April 18th, 2013 6 / 34

Page 17: Biostatistics 602 - Statistical Inference Lecture 25 ... · Bayesian Tests • Hypothesis testing problems can be formulated in a Bayesian model • Bayesian model includes • Sampling

..........

.....

......

.....

.....

.....

......

.....

.....

.....

......

.....

.....

.....

......

.....

......

.....

.....

.

. . . .Recap

. . . . .Bayesian Tests

. . . . .Bayesian Intervals

. . .P1

. . . . . . . .P2

. . . .P3

. . . .P4

Bayesian Tests

• Hypothesis testing problems can be formulated in a Bayesian model• Bayesian model includes

• Sampling distribution f(x|θ)• Prior distribution π(θ)

• Bayesian hypothesis testing is based on the posterior probability• In Frequentist’s framework, posterior probability cannot be calculated.• In Bayesian framework, the probability of H0 and H1 can be calculated

• Pr(θ ∈ Ω0|x) = Pr(H0 is true)

• Pr(θ ∈ Ωc0|x) = Pr(H1 is true)

• Rejection region can be determined directly based on the posteriorprobability

Hyun Min Kang Biostatistics 602 - Lecture 25 April 18th, 2013 6 / 34

Page 18: Biostatistics 602 - Statistical Inference Lecture 25 ... · Bayesian Tests • Hypothesis testing problems can be formulated in a Bayesian model • Bayesian model includes • Sampling

..........

.....

......

.....

.....

.....

......

.....

.....

.....

......

.....

.....

.....

......

.....

......

.....

.....

.

. . . .Recap

. . . . .Bayesian Tests

. . . . .Bayesian Intervals

. . .P1

. . . . . . . .P2

. . . .P3

. . . .P4

Bayesian Tests

• Hypothesis testing problems can be formulated in a Bayesian model• Bayesian model includes

• Sampling distribution f(x|θ)• Prior distribution π(θ)

• Bayesian hypothesis testing is based on the posterior probability• In Frequentist’s framework, posterior probability cannot be calculated.• In Bayesian framework, the probability of H0 and H1 can be calculated

• Pr(θ ∈ Ω0|x) = Pr(H0 is true)• Pr(θ ∈ Ωc

0|x) = Pr(H1 is true)

• Rejection region can be determined directly based on the posteriorprobability

Hyun Min Kang Biostatistics 602 - Lecture 25 April 18th, 2013 6 / 34

Page 19: Biostatistics 602 - Statistical Inference Lecture 25 ... · Bayesian Tests • Hypothesis testing problems can be formulated in a Bayesian model • Bayesian model includes • Sampling

..........

.....

......

.....

.....

.....

......

.....

.....

.....

......

.....

.....

.....

......

.....

......

.....

.....

.

. . . .Recap

. . . . .Bayesian Tests

. . . . .Bayesian Intervals

. . .P1

. . . . . . . .P2

. . . .P3

. . . .P4

Bayesian Tests

• Hypothesis testing problems can be formulated in a Bayesian model• Bayesian model includes

• Sampling distribution f(x|θ)• Prior distribution π(θ)

• Bayesian hypothesis testing is based on the posterior probability• In Frequentist’s framework, posterior probability cannot be calculated.• In Bayesian framework, the probability of H0 and H1 can be calculated

• Pr(θ ∈ Ω0|x) = Pr(H0 is true)• Pr(θ ∈ Ωc

0|x) = Pr(H1 is true)• Rejection region can be determined directly based on the posterior

probability

Hyun Min Kang Biostatistics 602 - Lecture 25 April 18th, 2013 6 / 34

Page 20: Biostatistics 602 - Statistical Inference Lecture 25 ... · Bayesian Tests • Hypothesis testing problems can be formulated in a Bayesian model • Bayesian model includes • Sampling

..........

.....

......

.....

.....

.....

......

.....

.....

.....

......

.....

.....

.....

......

.....

......

.....

.....

.

. . . .Recap

. . . . .Bayesian Tests

. . . . .Bayesian Intervals

. . .P1

. . . . . . . .P2

. . . .P3

. . . .P4

Bayesian vs Frequentist Framework

.Frequentist’s Framework..

......

• θ is considered to be a fixed number

• Consequently, a hypothesis is either true of false• If θ ∈ Ω0, Pr(H0 is true|x) = 1 and Pr(H1 is true|x) = 0• If θ ∈ Ωc

0, Pr(H0 is true|x) = 0 and Pr(H1 is true|x) = 1

.Bayesian Framework..

......

• Pr(H0 is true|x) and Pr(H1 is true|x) are function of x, between 0and 1.

• These probabilities give useful information about the veracity of H0

and H1.

Hyun Min Kang Biostatistics 602 - Lecture 25 April 18th, 2013 7 / 34

Page 21: Biostatistics 602 - Statistical Inference Lecture 25 ... · Bayesian Tests • Hypothesis testing problems can be formulated in a Bayesian model • Bayesian model includes • Sampling

..........

.....

......

.....

.....

.....

......

.....

.....

.....

......

.....

.....

.....

......

.....

......

.....

.....

.

. . . .Recap

. . . . .Bayesian Tests

. . . . .Bayesian Intervals

. . .P1

. . . . . . . .P2

. . . .P3

. . . .P4

Bayesian vs Frequentist Framework

.Frequentist’s Framework..

......

• θ is considered to be a fixed number• Consequently, a hypothesis is either true of false

• If θ ∈ Ω0, Pr(H0 is true|x) = 1 and Pr(H1 is true|x) = 0• If θ ∈ Ωc

0, Pr(H0 is true|x) = 0 and Pr(H1 is true|x) = 1

.Bayesian Framework..

......

• Pr(H0 is true|x) and Pr(H1 is true|x) are function of x, between 0and 1.

• These probabilities give useful information about the veracity of H0

and H1.

Hyun Min Kang Biostatistics 602 - Lecture 25 April 18th, 2013 7 / 34

Page 22: Biostatistics 602 - Statistical Inference Lecture 25 ... · Bayesian Tests • Hypothesis testing problems can be formulated in a Bayesian model • Bayesian model includes • Sampling

..........

.....

......

.....

.....

.....

......

.....

.....

.....

......

.....

.....

.....

......

.....

......

.....

.....

.

. . . .Recap

. . . . .Bayesian Tests

. . . . .Bayesian Intervals

. . .P1

. . . . . . . .P2

. . . .P3

. . . .P4

Bayesian vs Frequentist Framework

.Frequentist’s Framework..

......

• θ is considered to be a fixed number• Consequently, a hypothesis is either true of false

• If θ ∈ Ω0, Pr(H0 is true|x) = 1 and Pr(H1 is true|x) = 0• If θ ∈ Ωc

0, Pr(H0 is true|x) = 0 and Pr(H1 is true|x) = 1

.Bayesian Framework..

......

• Pr(H0 is true|x) and Pr(H1 is true|x) are function of x, between 0and 1.

• These probabilities give useful information about the veracity of H0

and H1.

Hyun Min Kang Biostatistics 602 - Lecture 25 April 18th, 2013 7 / 34

Page 23: Biostatistics 602 - Statistical Inference Lecture 25 ... · Bayesian Tests • Hypothesis testing problems can be formulated in a Bayesian model • Bayesian model includes • Sampling

..........

.....

......

.....

.....

.....

......

.....

.....

.....

......

.....

.....

.....

......

.....

......

.....

.....

.

. . . .Recap

. . . . .Bayesian Tests

. . . . .Bayesian Intervals

. . .P1

. . . . . . . .P2

. . . .P3

. . . .P4

Bayesian vs Frequentist Framework

.Frequentist’s Framework..

......

• θ is considered to be a fixed number• Consequently, a hypothesis is either true of false

• If θ ∈ Ω0, Pr(H0 is true|x) = 1 and Pr(H1 is true|x) = 0• If θ ∈ Ωc

0, Pr(H0 is true|x) = 0 and Pr(H1 is true|x) = 1

.Bayesian Framework..

......

• Pr(H0 is true|x) and Pr(H1 is true|x) are function of x, between 0and 1.

• These probabilities give useful information about the veracity of H0

and H1.

Hyun Min Kang Biostatistics 602 - Lecture 25 April 18th, 2013 7 / 34

Page 24: Biostatistics 602 - Statistical Inference Lecture 25 ... · Bayesian Tests • Hypothesis testing problems can be formulated in a Bayesian model • Bayesian model includes • Sampling

..........

.....

......

.....

.....

.....

......

.....

.....

.....

......

.....

.....

.....

......

.....

......

.....

.....

.

. . . .Recap

. . . . .Bayesian Tests

. . . . .Bayesian Intervals

. . .P1

. . . . . . . .P2

. . . .P3

. . . .P4

Examples of Bayesian hypothesis testing procedure

.A neutral test between H0 and H1..

......

• Accept H0 is Pr(θ ∈ Ω0|x) ≥ Pr(θ ∈ Ωc0|x)

• Reject H0 is Pr(θ ∈ Ω0|x) < Pr(θ ∈ Ωc0|x)

• In other words, the rejection region is x : Pr(θ ∈ Ωc0|x) > 1

2

.A more conservative (smaller size) test in rejecting H0..

......

• Reject H0 is Pr(θ ∈ Ωc0|x) > 0.99

• Accept H0 is Pr(θ ∈ Ωc0|x) ≤ 0.99

Hyun Min Kang Biostatistics 602 - Lecture 25 April 18th, 2013 8 / 34

Page 25: Biostatistics 602 - Statistical Inference Lecture 25 ... · Bayesian Tests • Hypothesis testing problems can be formulated in a Bayesian model • Bayesian model includes • Sampling

..........

.....

......

.....

.....

.....

......

.....

.....

.....

......

.....

.....

.....

......

.....

......

.....

.....

.

. . . .Recap

. . . . .Bayesian Tests

. . . . .Bayesian Intervals

. . .P1

. . . . . . . .P2

. . . .P3

. . . .P4

Examples of Bayesian hypothesis testing procedure

.A neutral test between H0 and H1..

......

• Accept H0 is Pr(θ ∈ Ω0|x) ≥ Pr(θ ∈ Ωc0|x)

• Reject H0 is Pr(θ ∈ Ω0|x) < Pr(θ ∈ Ωc0|x)

• In other words, the rejection region is x : Pr(θ ∈ Ωc0|x) > 1

2

.A more conservative (smaller size) test in rejecting H0..

......

• Reject H0 is Pr(θ ∈ Ωc0|x) > 0.99

• Accept H0 is Pr(θ ∈ Ωc0|x) ≤ 0.99

Hyun Min Kang Biostatistics 602 - Lecture 25 April 18th, 2013 8 / 34

Page 26: Biostatistics 602 - Statistical Inference Lecture 25 ... · Bayesian Tests • Hypothesis testing problems can be formulated in a Bayesian model • Bayesian model includes • Sampling

..........

.....

......

.....

.....

.....

......

.....

.....

.....

......

.....

.....

.....

......

.....

......

.....

.....

.

. . . .Recap

. . . . .Bayesian Tests

. . . . .Bayesian Intervals

. . .P1

. . . . . . . .P2

. . . .P3

. . . .P4

Examples of Bayesian hypothesis testing procedure

.A neutral test between H0 and H1..

......

• Accept H0 is Pr(θ ∈ Ω0|x) ≥ Pr(θ ∈ Ωc0|x)

• Reject H0 is Pr(θ ∈ Ω0|x) < Pr(θ ∈ Ωc0|x)

• In other words, the rejection region is x : Pr(θ ∈ Ωc0|x) > 1

2

.A more conservative (smaller size) test in rejecting H0..

......

• Reject H0 is Pr(θ ∈ Ωc0|x) > 0.99

• Accept H0 is Pr(θ ∈ Ωc0|x) ≤ 0.99

Hyun Min Kang Biostatistics 602 - Lecture 25 April 18th, 2013 8 / 34

Page 27: Biostatistics 602 - Statistical Inference Lecture 25 ... · Bayesian Tests • Hypothesis testing problems can be formulated in a Bayesian model • Bayesian model includes • Sampling

..........

.....

......

.....

.....

.....

......

.....

.....

.....

......

.....

.....

.....

......

.....

......

.....

.....

.

. . . .Recap

. . . . .Bayesian Tests

. . . . .Bayesian Intervals

. . .P1

. . . . . . . .P2

. . . .P3

. . . .P4

Examples of Bayesian hypothesis testing procedure

.A neutral test between H0 and H1..

......

• Accept H0 is Pr(θ ∈ Ω0|x) ≥ Pr(θ ∈ Ωc0|x)

• Reject H0 is Pr(θ ∈ Ω0|x) < Pr(θ ∈ Ωc0|x)

• In other words, the rejection region is x : Pr(θ ∈ Ωc0|x) > 1

2

.A more conservative (smaller size) test in rejecting H0..

......

• Reject H0 is Pr(θ ∈ Ωc0|x) > 0.99

• Accept H0 is Pr(θ ∈ Ωc0|x) ≤ 0.99

Hyun Min Kang Biostatistics 602 - Lecture 25 April 18th, 2013 8 / 34

Page 28: Biostatistics 602 - Statistical Inference Lecture 25 ... · Bayesian Tests • Hypothesis testing problems can be formulated in a Bayesian model • Bayesian model includes • Sampling

..........

.....

......

.....

.....

.....

......

.....

.....

.....

......

.....

.....

.....

......

.....

......

.....

.....

.

. . . .Recap

. . . . .Bayesian Tests

. . . . .Bayesian Intervals

. . .P1

. . . . . . . .P2

. . . .P3

. . . .P4

Example: Normal Bayesian Test.Problem..

......

Let X1, · · · ,Xn be iid samples N (θ, σ2) and let the prior distribution of θbe N (µ, τ r), where σ2, µ, and τ2 are known.

Construct a Bayesian testrejecting H0 if and only if Pr(θ ∈ Ω0|x) < Pr(θ ∈ Ωc

0|x)

.Solution..

......

Consider testing H0 : θ ≤ θ0 versus H1 : θ > θ0. From previous lectures,the posterior is

π(θ|x) ∼ N(

nτ2x + σ2µ

nτ2 + σ2,

σ2τ2

nτ2 + σ2

)We will reject H0 if and only if

Pr(θ ∈ Ω0|x) = Pr(θ ≤ θ0|x) <1

2

Hyun Min Kang Biostatistics 602 - Lecture 25 April 18th, 2013 9 / 34

Page 29: Biostatistics 602 - Statistical Inference Lecture 25 ... · Bayesian Tests • Hypothesis testing problems can be formulated in a Bayesian model • Bayesian model includes • Sampling

..........

.....

......

.....

.....

.....

......

.....

.....

.....

......

.....

.....

.....

......

.....

......

.....

.....

.

. . . .Recap

. . . . .Bayesian Tests

. . . . .Bayesian Intervals

. . .P1

. . . . . . . .P2

. . . .P3

. . . .P4

Example: Normal Bayesian Test.Problem..

......

Let X1, · · · ,Xn be iid samples N (θ, σ2) and let the prior distribution of θbe N (µ, τ r), where σ2, µ, and τ2 are known. Construct a Bayesian testrejecting H0 if and only if Pr(θ ∈ Ω0|x) < Pr(θ ∈ Ωc

0|x)

.Solution..

......

Consider testing H0 : θ ≤ θ0 versus H1 : θ > θ0. From previous lectures,the posterior is

π(θ|x) ∼ N(

nτ2x + σ2µ

nτ2 + σ2,

σ2τ2

nτ2 + σ2

)We will reject H0 if and only if

Pr(θ ∈ Ω0|x) = Pr(θ ≤ θ0|x) <1

2

Hyun Min Kang Biostatistics 602 - Lecture 25 April 18th, 2013 9 / 34

Page 30: Biostatistics 602 - Statistical Inference Lecture 25 ... · Bayesian Tests • Hypothesis testing problems can be formulated in a Bayesian model • Bayesian model includes • Sampling

..........

.....

......

.....

.....

.....

......

.....

.....

.....

......

.....

.....

.....

......

.....

......

.....

.....

.

. . . .Recap

. . . . .Bayesian Tests

. . . . .Bayesian Intervals

. . .P1

. . . . . . . .P2

. . . .P3

. . . .P4

Example: Normal Bayesian Test.Problem..

......

Let X1, · · · ,Xn be iid samples N (θ, σ2) and let the prior distribution of θbe N (µ, τ r), where σ2, µ, and τ2 are known. Construct a Bayesian testrejecting H0 if and only if Pr(θ ∈ Ω0|x) < Pr(θ ∈ Ωc

0|x)

.Solution..

......

Consider testing H0 : θ ≤ θ0 versus H1 : θ > θ0. From previous lectures,the posterior is

π(θ|x) ∼ N(

nτ2x + σ2µ

nτ2 + σ2,

σ2τ2

nτ2 + σ2

)We will reject H0 if and only if

Pr(θ ∈ Ω0|x) = Pr(θ ≤ θ0|x) <1

2

Hyun Min Kang Biostatistics 602 - Lecture 25 April 18th, 2013 9 / 34

Page 31: Biostatistics 602 - Statistical Inference Lecture 25 ... · Bayesian Tests • Hypothesis testing problems can be formulated in a Bayesian model • Bayesian model includes • Sampling

..........

.....

......

.....

.....

.....

......

.....

.....

.....

......

.....

.....

.....

......

.....

......

.....

.....

.

. . . .Recap

. . . . .Bayesian Tests

. . . . .Bayesian Intervals

. . .P1

. . . . . . . .P2

. . . .P3

. . . .P4

Example: Normal Bayesian Test.Problem..

......

Let X1, · · · ,Xn be iid samples N (θ, σ2) and let the prior distribution of θbe N (µ, τ r), where σ2, µ, and τ2 are known. Construct a Bayesian testrejecting H0 if and only if Pr(θ ∈ Ω0|x) < Pr(θ ∈ Ωc

0|x)

.Solution..

......

Consider testing H0 : θ ≤ θ0 versus H1 : θ > θ0. From previous lectures,the posterior is

π(θ|x) ∼ N(

nτ2x + σ2µ

nτ2 + σ2,

σ2τ2

nτ2 + σ2

)

We will reject H0 if and only ifPr(θ ∈ Ω0|x) = Pr(θ ≤ θ0|x) <

1

2

Hyun Min Kang Biostatistics 602 - Lecture 25 April 18th, 2013 9 / 34

Page 32: Biostatistics 602 - Statistical Inference Lecture 25 ... · Bayesian Tests • Hypothesis testing problems can be formulated in a Bayesian model • Bayesian model includes • Sampling

..........

.....

......

.....

.....

.....

......

.....

.....

.....

......

.....

.....

.....

......

.....

......

.....

.....

.

. . . .Recap

. . . . .Bayesian Tests

. . . . .Bayesian Intervals

. . .P1

. . . . . . . .P2

. . . .P3

. . . .P4

Example: Normal Bayesian Test.Problem..

......

Let X1, · · · ,Xn be iid samples N (θ, σ2) and let the prior distribution of θbe N (µ, τ r), where σ2, µ, and τ2 are known. Construct a Bayesian testrejecting H0 if and only if Pr(θ ∈ Ω0|x) < Pr(θ ∈ Ωc

0|x)

.Solution..

......

Consider testing H0 : θ ≤ θ0 versus H1 : θ > θ0. From previous lectures,the posterior is

π(θ|x) ∼ N(

nτ2x + σ2µ

nτ2 + σ2,

σ2τ2

nτ2 + σ2

)We will reject H0 if and only if

Pr(θ ∈ Ω0|x) = Pr(θ ≤ θ0|x) <1

2

Hyun Min Kang Biostatistics 602 - Lecture 25 April 18th, 2013 9 / 34

Page 33: Biostatistics 602 - Statistical Inference Lecture 25 ... · Bayesian Tests • Hypothesis testing problems can be formulated in a Bayesian model • Bayesian model includes • Sampling

..........

.....

......

.....

.....

.....

......

.....

.....

.....

......

.....

.....

.....

......

.....

......

.....

.....

.

. . . .Recap

. . . . .Bayesian Tests

. . . . .Bayesian Intervals

. . .P1

. . . . . . . .P2

. . . .P3

. . . .P4

Solution (cont’d)

Because π(θ|x) is symmetric, this is true if and only if the mean for π(θ|x)is less than or equal to θ0. Therefore, H0 will be rejected if

nτ2x + σ2µ

nτ2 + σ2< θ0

x < θ0 +σ2(θ0 − µ)

nτ2

Hyun Min Kang Biostatistics 602 - Lecture 25 April 18th, 2013 10 / 34

Page 34: Biostatistics 602 - Statistical Inference Lecture 25 ... · Bayesian Tests • Hypothesis testing problems can be formulated in a Bayesian model • Bayesian model includes • Sampling

..........

.....

......

.....

.....

.....

......

.....

.....

.....

......

.....

.....

.....

......

.....

......

.....

.....

.

. . . .Recap

. . . . .Bayesian Tests

. . . . .Bayesian Intervals

. . .P1

. . . . . . . .P2

. . . .P3

. . . .P4

Solution (cont’d)

Because π(θ|x) is symmetric, this is true if and only if the mean for π(θ|x)is less than or equal to θ0. Therefore, H0 will be rejected if

nτ2x + σ2µ

nτ2 + σ2< θ0

x < θ0 +σ2(θ0 − µ)

nτ2

Hyun Min Kang Biostatistics 602 - Lecture 25 April 18th, 2013 10 / 34

Page 35: Biostatistics 602 - Statistical Inference Lecture 25 ... · Bayesian Tests • Hypothesis testing problems can be formulated in a Bayesian model • Bayesian model includes • Sampling

..........

.....

......

.....

.....

.....

......

.....

.....

.....

......

.....

.....

.....

......

.....

......

.....

.....

.

. . . .Recap

. . . . .Bayesian Tests

. . . . .Bayesian Intervals

. . .P1

. . . . . . . .P2

. . . .P3

. . . .P4

Solution (cont’d)

Because π(θ|x) is symmetric, this is true if and only if the mean for π(θ|x)is less than or equal to θ0. Therefore, H0 will be rejected if

nτ2x + σ2µ

nτ2 + σ2< θ0

x < θ0 +σ2(θ0 − µ)

nτ2

Hyun Min Kang Biostatistics 602 - Lecture 25 April 18th, 2013 10 / 34

Page 36: Biostatistics 602 - Statistical Inference Lecture 25 ... · Bayesian Tests • Hypothesis testing problems can be formulated in a Bayesian model • Bayesian model includes • Sampling

..........

.....

......

.....

.....

.....

......

.....

.....

.....

......

.....

.....

.....

......

.....

......

.....

.....

.

. . . .Recap

. . . . .Bayesian Tests

. . . . .Bayesian Intervals

. . .P1

. . . . . . . .P2

. . . .P3

. . . .P4

Confidence interval and the parameter

.Frequentist’s view of intervals..

......

• We have carefully said that the interval covers the parameter• not that the parameter is inside the interval, on purpose.• The random quantity is the interval, not the parameter

.Example..

......

• A 95% confidence interval for θ is .262 ≤ θ ≤ 1.184

• ”The probability that θ is in the interval [.262,1.184] is 95%” :Incorrect, because the parameter is assumed fixed

• Formally, the interval [.262,1.184] is one of the possible realizedvalues of the random intervals (depending on the observed data)

Hyun Min Kang Biostatistics 602 - Lecture 25 April 18th, 2013 11 / 34

Page 37: Biostatistics 602 - Statistical Inference Lecture 25 ... · Bayesian Tests • Hypothesis testing problems can be formulated in a Bayesian model • Bayesian model includes • Sampling

..........

.....

......

.....

.....

.....

......

.....

.....

.....

......

.....

.....

.....

......

.....

......

.....

.....

.

. . . .Recap

. . . . .Bayesian Tests

. . . . .Bayesian Intervals

. . .P1

. . . . . . . .P2

. . . .P3

. . . .P4

Confidence interval and the parameter

.Frequentist’s view of intervals..

......

• We have carefully said that the interval covers the parameter

• not that the parameter is inside the interval, on purpose.• The random quantity is the interval, not the parameter

.Example..

......

• A 95% confidence interval for θ is .262 ≤ θ ≤ 1.184

• ”The probability that θ is in the interval [.262,1.184] is 95%” :Incorrect, because the parameter is assumed fixed

• Formally, the interval [.262,1.184] is one of the possible realizedvalues of the random intervals (depending on the observed data)

Hyun Min Kang Biostatistics 602 - Lecture 25 April 18th, 2013 11 / 34

Page 38: Biostatistics 602 - Statistical Inference Lecture 25 ... · Bayesian Tests • Hypothesis testing problems can be formulated in a Bayesian model • Bayesian model includes • Sampling

..........

.....

......

.....

.....

.....

......

.....

.....

.....

......

.....

.....

.....

......

.....

......

.....

.....

.

. . . .Recap

. . . . .Bayesian Tests

. . . . .Bayesian Intervals

. . .P1

. . . . . . . .P2

. . . .P3

. . . .P4

Confidence interval and the parameter

.Frequentist’s view of intervals..

......

• We have carefully said that the interval covers the parameter• not that the parameter is inside the interval, on purpose.

• The random quantity is the interval, not the parameter

.Example..

......

• A 95% confidence interval for θ is .262 ≤ θ ≤ 1.184

• ”The probability that θ is in the interval [.262,1.184] is 95%” :Incorrect, because the parameter is assumed fixed

• Formally, the interval [.262,1.184] is one of the possible realizedvalues of the random intervals (depending on the observed data)

Hyun Min Kang Biostatistics 602 - Lecture 25 April 18th, 2013 11 / 34

Page 39: Biostatistics 602 - Statistical Inference Lecture 25 ... · Bayesian Tests • Hypothesis testing problems can be formulated in a Bayesian model • Bayesian model includes • Sampling

..........

.....

......

.....

.....

.....

......

.....

.....

.....

......

.....

.....

.....

......

.....

......

.....

.....

.

. . . .Recap

. . . . .Bayesian Tests

. . . . .Bayesian Intervals

. . .P1

. . . . . . . .P2

. . . .P3

. . . .P4

Confidence interval and the parameter

.Frequentist’s view of intervals..

......

• We have carefully said that the interval covers the parameter• not that the parameter is inside the interval, on purpose.• The random quantity is the interval, not the parameter

.Example..

......

• A 95% confidence interval for θ is .262 ≤ θ ≤ 1.184

• ”The probability that θ is in the interval [.262,1.184] is 95%” :Incorrect, because the parameter is assumed fixed

• Formally, the interval [.262,1.184] is one of the possible realizedvalues of the random intervals (depending on the observed data)

Hyun Min Kang Biostatistics 602 - Lecture 25 April 18th, 2013 11 / 34

Page 40: Biostatistics 602 - Statistical Inference Lecture 25 ... · Bayesian Tests • Hypothesis testing problems can be formulated in a Bayesian model • Bayesian model includes • Sampling

..........

.....

......

.....

.....

.....

......

.....

.....

.....

......

.....

.....

.....

......

.....

......

.....

.....

.

. . . .Recap

. . . . .Bayesian Tests

. . . . .Bayesian Intervals

. . .P1

. . . . . . . .P2

. . . .P3

. . . .P4

Confidence interval and the parameter

.Frequentist’s view of intervals..

......

• We have carefully said that the interval covers the parameter• not that the parameter is inside the interval, on purpose.• The random quantity is the interval, not the parameter

.Example..

......

• A 95% confidence interval for θ is .262 ≤ θ ≤ 1.184

• ”The probability that θ is in the interval [.262,1.184] is 95%” :Incorrect, because the parameter is assumed fixed

• Formally, the interval [.262,1.184] is one of the possible realizedvalues of the random intervals (depending on the observed data)

Hyun Min Kang Biostatistics 602 - Lecture 25 April 18th, 2013 11 / 34

Page 41: Biostatistics 602 - Statistical Inference Lecture 25 ... · Bayesian Tests • Hypothesis testing problems can be formulated in a Bayesian model • Bayesian model includes • Sampling

..........

.....

......

.....

.....

.....

......

.....

.....

.....

......

.....

.....

.....

......

.....

......

.....

.....

.

. . . .Recap

. . . . .Bayesian Tests

. . . . .Bayesian Intervals

. . .P1

. . . . . . . .P2

. . . .P3

. . . .P4

Confidence interval and the parameter

.Frequentist’s view of intervals..

......

• We have carefully said that the interval covers the parameter• not that the parameter is inside the interval, on purpose.• The random quantity is the interval, not the parameter

.Example..

......

• A 95% confidence interval for θ is .262 ≤ θ ≤ 1.184

• ”The probability that θ is in the interval [.262,1.184] is 95%” :Incorrect, because the parameter is assumed fixed

• Formally, the interval [.262,1.184] is one of the possible realizedvalues of the random intervals (depending on the observed data)

Hyun Min Kang Biostatistics 602 - Lecture 25 April 18th, 2013 11 / 34

Page 42: Biostatistics 602 - Statistical Inference Lecture 25 ... · Bayesian Tests • Hypothesis testing problems can be formulated in a Bayesian model • Bayesian model includes • Sampling

..........

.....

......

.....

.....

.....

......

.....

.....

.....

......

.....

.....

.....

......

.....

......

.....

.....

.

. . . .Recap

. . . . .Bayesian Tests

. . . . .Bayesian Intervals

. . .P1

. . . . . . . .P2

. . . .P3

. . . .P4

Confidence interval and the parameter

.Frequentist’s view of intervals..

......

• We have carefully said that the interval covers the parameter• not that the parameter is inside the interval, on purpose.• The random quantity is the interval, not the parameter

.Example..

......

• A 95% confidence interval for θ is .262 ≤ θ ≤ 1.184

• ”The probability that θ is in the interval [.262,1.184] is 95%” :Incorrect, because the parameter is assumed fixed

• Formally, the interval [.262,1.184] is one of the possible realizedvalues of the random intervals (depending on the observed data)

Hyun Min Kang Biostatistics 602 - Lecture 25 April 18th, 2013 11 / 34

Page 43: Biostatistics 602 - Statistical Inference Lecture 25 ... · Bayesian Tests • Hypothesis testing problems can be formulated in a Bayesian model • Bayesian model includes • Sampling

..........

.....

......

.....

.....

.....

......

.....

.....

.....

......

.....

.....

.....

......

.....

......

.....

.....

.

. . . .Recap

. . . . .Bayesian Tests

. . . . .Bayesian Intervals

. . .P1

. . . . . . . .P2

. . . .P3

. . . .P4

Bayesian interpretation of intervals

• Bayesian setup allows us to say that θ is inside [.262, 1.184] withsome probability.

• Under Bayesian model, θ is a random variable with a probabilitydistribution.

• All Bayesian claims of coverage are made with respect to theposterior distribution of the parameter.

Hyun Min Kang Biostatistics 602 - Lecture 25 April 18th, 2013 12 / 34

Page 44: Biostatistics 602 - Statistical Inference Lecture 25 ... · Bayesian Tests • Hypothesis testing problems can be formulated in a Bayesian model • Bayesian model includes • Sampling

..........

.....

......

.....

.....

.....

......

.....

.....

.....

......

.....

.....

.....

......

.....

......

.....

.....

.

. . . .Recap

. . . . .Bayesian Tests

. . . . .Bayesian Intervals

. . .P1

. . . . . . . .P2

. . . .P3

. . . .P4

Bayesian interpretation of intervals

• Bayesian setup allows us to say that θ is inside [.262, 1.184] withsome probability.

• Under Bayesian model, θ is a random variable with a probabilitydistribution.

• All Bayesian claims of coverage are made with respect to theposterior distribution of the parameter.

Hyun Min Kang Biostatistics 602 - Lecture 25 April 18th, 2013 12 / 34

Page 45: Biostatistics 602 - Statistical Inference Lecture 25 ... · Bayesian Tests • Hypothesis testing problems can be formulated in a Bayesian model • Bayesian model includes • Sampling

..........

.....

......

.....

.....

.....

......

.....

.....

.....

......

.....

.....

.....

......

.....

......

.....

.....

.

. . . .Recap

. . . . .Bayesian Tests

. . . . .Bayesian Intervals

. . .P1

. . . . . . . .P2

. . . .P3

. . . .P4

Bayesian interpretation of intervals

• Bayesian setup allows us to say that θ is inside [.262, 1.184] withsome probability.

• Under Bayesian model, θ is a random variable with a probabilitydistribution.

• All Bayesian claims of coverage are made with respect to theposterior distribution of the parameter.

Hyun Min Kang Biostatistics 602 - Lecture 25 April 18th, 2013 12 / 34

Page 46: Biostatistics 602 - Statistical Inference Lecture 25 ... · Bayesian Tests • Hypothesis testing problems can be formulated in a Bayesian model • Bayesian model includes • Sampling

..........

.....

......

.....

.....

.....

......

.....

.....

.....

......

.....

.....

.....

......

.....

......

.....

.....

.

. . . .Recap

. . . . .Bayesian Tests

. . . . .Bayesian Intervals

. . .P1

. . . . . . . .P2

. . . .P3

. . . .P4

Credible sets

• To distinguish Bayesian estimates of coverage, we use credible setsrather than confidence sets

• If π(θ|x) is a posterior distribution, for any set A ⊂ Ω• The credible probability of A is Pr(θ ∈ A|x) =

∫A π(θ|x)dθ

• and A is a credible set (or creditable interval) for θ.• Both the interpretation and construction of the Bayes credible set are

more straightforward than those of a classical confidence set, but withadditional assumptions (for Bayesian framework).

Hyun Min Kang Biostatistics 602 - Lecture 25 April 18th, 2013 13 / 34

Page 47: Biostatistics 602 - Statistical Inference Lecture 25 ... · Bayesian Tests • Hypothesis testing problems can be formulated in a Bayesian model • Bayesian model includes • Sampling

..........

.....

......

.....

.....

.....

......

.....

.....

.....

......

.....

.....

.....

......

.....

......

.....

.....

.

. . . .Recap

. . . . .Bayesian Tests

. . . . .Bayesian Intervals

. . .P1

. . . . . . . .P2

. . . .P3

. . . .P4

Credible sets

• To distinguish Bayesian estimates of coverage, we use credible setsrather than confidence sets

• If π(θ|x) is a posterior distribution, for any set A ⊂ Ω

• The credible probability of A is Pr(θ ∈ A|x) =∫

A π(θ|x)dθ• and A is a credible set (or creditable interval) for θ.

• Both the interpretation and construction of the Bayes credible set aremore straightforward than those of a classical confidence set, but withadditional assumptions (for Bayesian framework).

Hyun Min Kang Biostatistics 602 - Lecture 25 April 18th, 2013 13 / 34

Page 48: Biostatistics 602 - Statistical Inference Lecture 25 ... · Bayesian Tests • Hypothesis testing problems can be formulated in a Bayesian model • Bayesian model includes • Sampling

..........

.....

......

.....

.....

.....

......

.....

.....

.....

......

.....

.....

.....

......

.....

......

.....

.....

.

. . . .Recap

. . . . .Bayesian Tests

. . . . .Bayesian Intervals

. . .P1

. . . . . . . .P2

. . . .P3

. . . .P4

Credible sets

• To distinguish Bayesian estimates of coverage, we use credible setsrather than confidence sets

• If π(θ|x) is a posterior distribution, for any set A ⊂ Ω• The credible probability of A is Pr(θ ∈ A|x) =

∫A π(θ|x)dθ

• and A is a credible set (or creditable interval) for θ.• Both the interpretation and construction of the Bayes credible set are

more straightforward than those of a classical confidence set, but withadditional assumptions (for Bayesian framework).

Hyun Min Kang Biostatistics 602 - Lecture 25 April 18th, 2013 13 / 34

Page 49: Biostatistics 602 - Statistical Inference Lecture 25 ... · Bayesian Tests • Hypothesis testing problems can be formulated in a Bayesian model • Bayesian model includes • Sampling

..........

.....

......

.....

.....

.....

......

.....

.....

.....

......

.....

.....

.....

......

.....

......

.....

.....

.

. . . .Recap

. . . . .Bayesian Tests

. . . . .Bayesian Intervals

. . .P1

. . . . . . . .P2

. . . .P3

. . . .P4

Credible sets

• To distinguish Bayesian estimates of coverage, we use credible setsrather than confidence sets

• If π(θ|x) is a posterior distribution, for any set A ⊂ Ω• The credible probability of A is Pr(θ ∈ A|x) =

∫A π(θ|x)dθ

• and A is a credible set (or creditable interval) for θ.

• Both the interpretation and construction of the Bayes credible set aremore straightforward than those of a classical confidence set, but withadditional assumptions (for Bayesian framework).

Hyun Min Kang Biostatistics 602 - Lecture 25 April 18th, 2013 13 / 34

Page 50: Biostatistics 602 - Statistical Inference Lecture 25 ... · Bayesian Tests • Hypothesis testing problems can be formulated in a Bayesian model • Bayesian model includes • Sampling

..........

.....

......

.....

.....

.....

......

.....

.....

.....

......

.....

.....

.....

......

.....

......

.....

.....

.

. . . .Recap

. . . . .Bayesian Tests

. . . . .Bayesian Intervals

. . .P1

. . . . . . . .P2

. . . .P3

. . . .P4

Credible sets

• To distinguish Bayesian estimates of coverage, we use credible setsrather than confidence sets

• If π(θ|x) is a posterior distribution, for any set A ⊂ Ω• The credible probability of A is Pr(θ ∈ A|x) =

∫A π(θ|x)dθ

• and A is a credible set (or creditable interval) for θ.• Both the interpretation and construction of the Bayes credible set are

more straightforward than those of a classical confidence set, but withadditional assumptions (for Bayesian framework).

Hyun Min Kang Biostatistics 602 - Lecture 25 April 18th, 2013 13 / 34

Page 51: Biostatistics 602 - Statistical Inference Lecture 25 ... · Bayesian Tests • Hypothesis testing problems can be formulated in a Bayesian model • Bayesian model includes • Sampling

..........

.....

......

.....

.....

.....

......

.....

.....

.....

......

.....

.....

.....

......

.....

......

.....

.....

.

. . . .Recap

. . . . .Bayesian Tests

. . . . .Bayesian Intervals

. . .P1

. . . . . . . .P2

. . . .P3

. . . .P4

Example: Possible credible set.Problem..

......Let X1, · · · ,Xn

i.i.d.∼ Poisson(λ) and assume that λ ∼ Gamma(a, b). Finda 90% credible set for λ.

.Solution..

......

The posterior pdf of λ becomesπ(λ|x) = Gamma

(a +

∑xi, [n + (1/b)]−1

)If we simply split the α equally between the upper and lower endpoints,

2(nb + 1)

b λ ∼ χ22(a+

∑xi)

(if a is an integer)

Therefore, a 1− α confidence interval isλ :

b2(nb + 1)

χ22(∑

xi+a),1−α/2 ≤ λ ≤ b2(nb + 1)

χ22(∑

xi+a),α/2

Hyun Min Kang Biostatistics 602 - Lecture 25 April 18th, 2013 14 / 34

Page 52: Biostatistics 602 - Statistical Inference Lecture 25 ... · Bayesian Tests • Hypothesis testing problems can be formulated in a Bayesian model • Bayesian model includes • Sampling

..........

.....

......

.....

.....

.....

......

.....

.....

.....

......

.....

.....

.....

......

.....

......

.....

.....

.

. . . .Recap

. . . . .Bayesian Tests

. . . . .Bayesian Intervals

. . .P1

. . . . . . . .P2

. . . .P3

. . . .P4

Example: Possible credible set.Problem..

......Let X1, · · · ,Xn

i.i.d.∼ Poisson(λ) and assume that λ ∼ Gamma(a, b). Finda 90% credible set for λ..Solution..

......

The posterior pdf of λ becomesπ(λ|x) = Gamma

(a +

∑xi, [n + (1/b)]−1

)

If we simply split the α equally between the upper and lower endpoints,2(nb + 1)

b λ ∼ χ22(a+

∑xi)

(if a is an integer)

Therefore, a 1− α confidence interval isλ :

b2(nb + 1)

χ22(∑

xi+a),1−α/2 ≤ λ ≤ b2(nb + 1)

χ22(∑

xi+a),α/2

Hyun Min Kang Biostatistics 602 - Lecture 25 April 18th, 2013 14 / 34

Page 53: Biostatistics 602 - Statistical Inference Lecture 25 ... · Bayesian Tests • Hypothesis testing problems can be formulated in a Bayesian model • Bayesian model includes • Sampling

..........

.....

......

.....

.....

.....

......

.....

.....

.....

......

.....

.....

.....

......

.....

......

.....

.....

.

. . . .Recap

. . . . .Bayesian Tests

. . . . .Bayesian Intervals

. . .P1

. . . . . . . .P2

. . . .P3

. . . .P4

Example: Possible credible set.Problem..

......Let X1, · · · ,Xn

i.i.d.∼ Poisson(λ) and assume that λ ∼ Gamma(a, b). Finda 90% credible set for λ..Solution..

......

The posterior pdf of λ becomesπ(λ|x) = Gamma

(a +

∑xi, [n + (1/b)]−1

)If we simply split the α equally between the upper and lower endpoints,

2(nb + 1)

b λ ∼ χ22(a+

∑xi)

(if a is an integer)

Therefore, a 1− α confidence interval isλ :

b2(nb + 1)

χ22(∑

xi+a),1−α/2 ≤ λ ≤ b2(nb + 1)

χ22(∑

xi+a),α/2

Hyun Min Kang Biostatistics 602 - Lecture 25 April 18th, 2013 14 / 34

Page 54: Biostatistics 602 - Statistical Inference Lecture 25 ... · Bayesian Tests • Hypothesis testing problems can be formulated in a Bayesian model • Bayesian model includes • Sampling

..........

.....

......

.....

.....

.....

......

.....

.....

.....

......

.....

.....

.....

......

.....

......

.....

.....

.

. . . .Recap

. . . . .Bayesian Tests

. . . . .Bayesian Intervals

. . .P1

. . . . . . . .P2

. . . .P3

. . . .P4

Example: Possible credible set.Problem..

......Let X1, · · · ,Xn

i.i.d.∼ Poisson(λ) and assume that λ ∼ Gamma(a, b). Finda 90% credible set for λ..Solution..

......

The posterior pdf of λ becomesπ(λ|x) = Gamma

(a +

∑xi, [n + (1/b)]−1

)If we simply split the α equally between the upper and lower endpoints,

2(nb + 1)

b λ ∼ χ22(a+

∑xi)

(if a is an integer)

Therefore, a 1− α confidence interval isλ :

b2(nb + 1)

χ22(∑

xi+a),1−α/2 ≤ λ ≤ b2(nb + 1)

χ22(∑

xi+a),α/2

Hyun Min Kang Biostatistics 602 - Lecture 25 April 18th, 2013 14 / 34

Page 55: Biostatistics 602 - Statistical Inference Lecture 25 ... · Bayesian Tests • Hypothesis testing problems can be formulated in a Bayesian model • Bayesian model includes • Sampling

..........

.....

......

.....

.....

.....

......

.....

.....

.....

......

.....

.....

.....

......

.....

......

.....

.....

.

. . . .Recap

. . . . .Bayesian Tests

. . . . .Bayesian Intervals

. . .P1

. . . . . . . .P2

. . . .P3

. . . .P4

Example: Possible credible set.Problem..

......Let X1, · · · ,Xn

i.i.d.∼ Poisson(λ) and assume that λ ∼ Gamma(a, b). Finda 90% credible set for λ..Solution..

......

The posterior pdf of λ becomesπ(λ|x) = Gamma

(a +

∑xi, [n + (1/b)]−1

)If we simply split the α equally between the upper and lower endpoints,

2(nb + 1)

b λ ∼ χ22(a+

∑xi)

(if a is an integer)

Therefore, a 1− α confidence interval isλ :

b2(nb + 1)

χ22(∑

xi+a),1−α/2 ≤ λ ≤ b2(nb + 1)

χ22(∑

xi+a),α/2

Hyun Min Kang Biostatistics 602 - Lecture 25 April 18th, 2013 14 / 34

Page 56: Biostatistics 602 - Statistical Inference Lecture 25 ... · Bayesian Tests • Hypothesis testing problems can be formulated in a Bayesian model • Bayesian model includes • Sampling

..........

.....

......

.....

.....

.....

......

.....

.....

.....

......

.....

.....

.....

......

.....

......

.....

.....

.

. . . .Recap

. . . . .Bayesian Tests

. . . . .Bayesian Intervals

. . .P1

. . . . . . . .P2

. . . .P3

. . . .P4

Remark: Credible probability and coverage probability

• It is important not to confuse credible probability with coverageprobability

• Credible probabilities are the Bayes posterior probability, whichreflects the experimenter’s subjective beliefs, as expressed in the priordistribution.

• A Bayesian assertion of 90% coverage means that the experimenter,upon combining prior knowledge with data, is 90% sure of coverage

• Coverage probability reflects the uncertainty in the samplingprocedure, getting its probability from the objective mechanism ofrepeated experimental trials.

• A classical assertion of 90% coverage means that in a long sequence ofidentical trials, 90% of the realized confidence sets will cover the trueparameter.

Hyun Min Kang Biostatistics 602 - Lecture 25 April 18th, 2013 15 / 34

Page 57: Biostatistics 602 - Statistical Inference Lecture 25 ... · Bayesian Tests • Hypothesis testing problems can be formulated in a Bayesian model • Bayesian model includes • Sampling

..........

.....

......

.....

.....

.....

......

.....

.....

.....

......

.....

.....

.....

......

.....

......

.....

.....

.

. . . .Recap

. . . . .Bayesian Tests

. . . . .Bayesian Intervals

. . .P1

. . . . . . . .P2

. . . .P3

. . . .P4

Remark: Credible probability and coverage probability

• It is important not to confuse credible probability with coverageprobability

• Credible probabilities are the Bayes posterior probability, whichreflects the experimenter’s subjective beliefs, as expressed in the priordistribution.

• A Bayesian assertion of 90% coverage means that the experimenter,upon combining prior knowledge with data, is 90% sure of coverage

• Coverage probability reflects the uncertainty in the samplingprocedure, getting its probability from the objective mechanism ofrepeated experimental trials.

• A classical assertion of 90% coverage means that in a long sequence ofidentical trials, 90% of the realized confidence sets will cover the trueparameter.

Hyun Min Kang Biostatistics 602 - Lecture 25 April 18th, 2013 15 / 34

Page 58: Biostatistics 602 - Statistical Inference Lecture 25 ... · Bayesian Tests • Hypothesis testing problems can be formulated in a Bayesian model • Bayesian model includes • Sampling

..........

.....

......

.....

.....

.....

......

.....

.....

.....

......

.....

.....

.....

......

.....

......

.....

.....

.

. . . .Recap

. . . . .Bayesian Tests

. . . . .Bayesian Intervals

. . .P1

. . . . . . . .P2

. . . .P3

. . . .P4

Remark: Credible probability and coverage probability

• It is important not to confuse credible probability with coverageprobability

• Credible probabilities are the Bayes posterior probability, whichreflects the experimenter’s subjective beliefs, as expressed in the priordistribution.

• A Bayesian assertion of 90% coverage means that the experimenter,upon combining prior knowledge with data, is 90% sure of coverage

• Coverage probability reflects the uncertainty in the samplingprocedure, getting its probability from the objective mechanism ofrepeated experimental trials.

• A classical assertion of 90% coverage means that in a long sequence ofidentical trials, 90% of the realized confidence sets will cover the trueparameter.

Hyun Min Kang Biostatistics 602 - Lecture 25 April 18th, 2013 15 / 34

Page 59: Biostatistics 602 - Statistical Inference Lecture 25 ... · Bayesian Tests • Hypothesis testing problems can be formulated in a Bayesian model • Bayesian model includes • Sampling

..........

.....

......

.....

.....

.....

......

.....

.....

.....

......

.....

.....

.....

......

.....

......

.....

.....

.

. . . .Recap

. . . . .Bayesian Tests

. . . . .Bayesian Intervals

. . .P1

. . . . . . . .P2

. . . .P3

. . . .P4

Remark: Credible probability and coverage probability

• It is important not to confuse credible probability with coverageprobability

• Credible probabilities are the Bayes posterior probability, whichreflects the experimenter’s subjective beliefs, as expressed in the priordistribution.

• A Bayesian assertion of 90% coverage means that the experimenter,upon combining prior knowledge with data, is 90% sure of coverage

• Coverage probability reflects the uncertainty in the samplingprocedure, getting its probability from the objective mechanism ofrepeated experimental trials.

• A classical assertion of 90% coverage means that in a long sequence ofidentical trials, 90% of the realized confidence sets will cover the trueparameter.

Hyun Min Kang Biostatistics 602 - Lecture 25 April 18th, 2013 15 / 34

Page 60: Biostatistics 602 - Statistical Inference Lecture 25 ... · Bayesian Tests • Hypothesis testing problems can be formulated in a Bayesian model • Bayesian model includes • Sampling

..........

.....

......

.....

.....

.....

......

.....

.....

.....

......

.....

.....

.....

......

.....

......

.....

.....

.

. . . .Recap

. . . . .Bayesian Tests

. . . . .Bayesian Intervals

. . .P1

. . . . . . . .P2

. . . .P3

. . . .P4

Remark: Credible probability and coverage probability

• It is important not to confuse credible probability with coverageprobability

• Credible probabilities are the Bayes posterior probability, whichreflects the experimenter’s subjective beliefs, as expressed in the priordistribution.

• A Bayesian assertion of 90% coverage means that the experimenter,upon combining prior knowledge with data, is 90% sure of coverage

• Coverage probability reflects the uncertainty in the samplingprocedure, getting its probability from the objective mechanism ofrepeated experimental trials.

• A classical assertion of 90% coverage means that in a long sequence ofidentical trials, 90% of the realized confidence sets will cover the trueparameter.

Hyun Min Kang Biostatistics 602 - Lecture 25 April 18th, 2013 15 / 34

Page 61: Biostatistics 602 - Statistical Inference Lecture 25 ... · Bayesian Tests • Hypothesis testing problems can be formulated in a Bayesian model • Bayesian model includes • Sampling

..........

.....

......

.....

.....

.....

......

.....

.....

.....

......

.....

.....

.....

......

.....

......

.....

.....

.

. . . .Recap

. . . . .Bayesian Tests

. . . . .Bayesian Intervals

. . .P1

. . . . . . . .P2

. . . .P3

. . . .P4

Practice Problem 1 (from last lecture)

.Problem..

......

Suppose X1, · · · ,Xn are iid samples from f(x|θ) = θ exp(−θx). Supposethe prior distribution of θ is

π(θ) =1

Γ(α)βαθα−1e−θ/β

where α, β are known.

(a) Derive the posterior distribution of θ.(b) If we use the loss function L(θ, a) = (a − θ)2, what is the Bayes rule

estimator for θ?

Hyun Min Kang Biostatistics 602 - Lecture 25 April 18th, 2013 16 / 34

Page 62: Biostatistics 602 - Statistical Inference Lecture 25 ... · Bayesian Tests • Hypothesis testing problems can be formulated in a Bayesian model • Bayesian model includes • Sampling

..........

.....

......

.....

.....

.....

......

.....

.....

.....

......

.....

.....

.....

......

.....

......

.....

.....

.

. . . .Recap

. . . . .Bayesian Tests

. . . . .Bayesian Intervals

. . .P1

. . . . . . . .P2

. . . .P3

. . . .P4

Practice Problem 1 (from last lecture)

.Problem..

......

Suppose X1, · · · ,Xn are iid samples from f(x|θ) = θ exp(−θx). Supposethe prior distribution of θ is

π(θ) =1

Γ(α)βαθα−1e−θ/β

where α, β are known.(a) Derive the posterior distribution of θ.

(b) If we use the loss function L(θ, a) = (a − θ)2, what is the Bayes ruleestimator for θ?

Hyun Min Kang Biostatistics 602 - Lecture 25 April 18th, 2013 16 / 34

Page 63: Biostatistics 602 - Statistical Inference Lecture 25 ... · Bayesian Tests • Hypothesis testing problems can be formulated in a Bayesian model • Bayesian model includes • Sampling

..........

.....

......

.....

.....

.....

......

.....

.....

.....

......

.....

.....

.....

......

.....

......

.....

.....

.

. . . .Recap

. . . . .Bayesian Tests

. . . . .Bayesian Intervals

. . .P1

. . . . . . . .P2

. . . .P3

. . . .P4

Practice Problem 1 (from last lecture)

.Problem..

......

Suppose X1, · · · ,Xn are iid samples from f(x|θ) = θ exp(−θx). Supposethe prior distribution of θ is

π(θ) =1

Γ(α)βαθα−1e−θ/β

where α, β are known.(a) Derive the posterior distribution of θ.(b) If we use the loss function L(θ, a) = (a − θ)2, what is the Bayes rule

estimator for θ?

Hyun Min Kang Biostatistics 602 - Lecture 25 April 18th, 2013 16 / 34

Page 64: Biostatistics 602 - Statistical Inference Lecture 25 ... · Bayesian Tests • Hypothesis testing problems can be formulated in a Bayesian model • Bayesian model includes • Sampling

..........

.....

......

.....

.....

.....

......

.....

.....

.....

......

.....

.....

.....

......

.....

......

.....

.....

.

. . . .Recap

. . . . .Bayesian Tests

. . . . .Bayesian Intervals

. . .P1

. . . . . . . .P2

. . . .P3

. . . .P4

(a) Posterior distribution of θ

f(x, θ) = π(θ)f(x|θ)π(θ)

=1

Γ(α)βαθα−1e−θ/β

n∏i=1

[θ exp (−θxi)]

=1

Γ(α)βαθα−1e−θ/βθn exp

(−θ

n∑i=1

xi

)

=1

Γ(α)βαθα+n−1 exp

[−θ

(1/β +

n∑i=1

xi

)]

∝ Gamma(α+ n − 1,

1

β−1 +∑n

i=1 xi

)π(θ|x) = Gamma

(α+ n − 1,

1

β−1 +∑n

i=1 xi

)

Hyun Min Kang Biostatistics 602 - Lecture 25 April 18th, 2013 17 / 34

Page 65: Biostatistics 602 - Statistical Inference Lecture 25 ... · Bayesian Tests • Hypothesis testing problems can be formulated in a Bayesian model • Bayesian model includes • Sampling

..........

.....

......

.....

.....

.....

......

.....

.....

.....

......

.....

.....

.....

......

.....

......

.....

.....

.

. . . .Recap

. . . . .Bayesian Tests

. . . . .Bayesian Intervals

. . .P1

. . . . . . . .P2

. . . .P3

. . . .P4

(a) Posterior distribution of θ

f(x, θ) = π(θ)f(x|θ)π(θ)

=1

Γ(α)βαθα−1e−θ/β

n∏i=1

[θ exp (−θxi)]

=1

Γ(α)βαθα−1e−θ/βθn exp

(−θ

n∑i=1

xi

)

=1

Γ(α)βαθα+n−1 exp

[−θ

(1/β +

n∑i=1

xi

)]

∝ Gamma(α+ n − 1,

1

β−1 +∑n

i=1 xi

)π(θ|x) = Gamma

(α+ n − 1,

1

β−1 +∑n

i=1 xi

)

Hyun Min Kang Biostatistics 602 - Lecture 25 April 18th, 2013 17 / 34

Page 66: Biostatistics 602 - Statistical Inference Lecture 25 ... · Bayesian Tests • Hypothesis testing problems can be formulated in a Bayesian model • Bayesian model includes • Sampling

..........

.....

......

.....

.....

.....

......

.....

.....

.....

......

.....

.....

.....

......

.....

......

.....

.....

.

. . . .Recap

. . . . .Bayesian Tests

. . . . .Bayesian Intervals

. . .P1

. . . . . . . .P2

. . . .P3

. . . .P4

(a) Posterior distribution of θ

f(x, θ) = π(θ)f(x|θ)π(θ)

=1

Γ(α)βαθα−1e−θ/β

n∏i=1

[θ exp (−θxi)]

=1

Γ(α)βαθα−1e−θ/βθn exp

(−θ

n∑i=1

xi

)

=1

Γ(α)βαθα+n−1 exp

[−θ

(1/β +

n∑i=1

xi

)]

∝ Gamma(α+ n − 1,

1

β−1 +∑n

i=1 xi

)π(θ|x) = Gamma

(α+ n − 1,

1

β−1 +∑n

i=1 xi

)

Hyun Min Kang Biostatistics 602 - Lecture 25 April 18th, 2013 17 / 34

Page 67: Biostatistics 602 - Statistical Inference Lecture 25 ... · Bayesian Tests • Hypothesis testing problems can be formulated in a Bayesian model • Bayesian model includes • Sampling

..........

.....

......

.....

.....

.....

......

.....

.....

.....

......

.....

.....

.....

......

.....

......

.....

.....

.

. . . .Recap

. . . . .Bayesian Tests

. . . . .Bayesian Intervals

. . .P1

. . . . . . . .P2

. . . .P3

. . . .P4

(a) Posterior distribution of θ

f(x, θ) = π(θ)f(x|θ)π(θ)

=1

Γ(α)βαθα−1e−θ/β

n∏i=1

[θ exp (−θxi)]

=1

Γ(α)βαθα−1e−θ/βθn exp

(−θ

n∑i=1

xi

)

=1

Γ(α)βαθα+n−1 exp

[−θ

(1/β +

n∑i=1

xi

)]

∝ Gamma(α+ n − 1,

1

β−1 +∑n

i=1 xi

)

π(θ|x) = Gamma(α+ n − 1,

1

β−1 +∑n

i=1 xi

)

Hyun Min Kang Biostatistics 602 - Lecture 25 April 18th, 2013 17 / 34

Page 68: Biostatistics 602 - Statistical Inference Lecture 25 ... · Bayesian Tests • Hypothesis testing problems can be formulated in a Bayesian model • Bayesian model includes • Sampling

..........

.....

......

.....

.....

.....

......

.....

.....

.....

......

.....

.....

.....

......

.....

......

.....

.....

.

. . . .Recap

. . . . .Bayesian Tests

. . . . .Bayesian Intervals

. . .P1

. . . . . . . .P2

. . . .P3

. . . .P4

(a) Posterior distribution of θ

f(x, θ) = π(θ)f(x|θ)π(θ)

=1

Γ(α)βαθα−1e−θ/β

n∏i=1

[θ exp (−θxi)]

=1

Γ(α)βαθα−1e−θ/βθn exp

(−θ

n∑i=1

xi

)

=1

Γ(α)βαθα+n−1 exp

[−θ

(1/β +

n∑i=1

xi

)]

∝ Gamma(α+ n − 1,

1

β−1 +∑n

i=1 xi

)π(θ|x) = Gamma

(α+ n − 1,

1

β−1 +∑n

i=1 xi

)Hyun Min Kang Biostatistics 602 - Lecture 25 April 18th, 2013 17 / 34

Page 69: Biostatistics 602 - Statistical Inference Lecture 25 ... · Bayesian Tests • Hypothesis testing problems can be formulated in a Bayesian model • Bayesian model includes • Sampling

..........

.....

......

.....

.....

.....

......

.....

.....

.....

......

.....

.....

.....

......

.....

......

.....

.....

.

. . . .Recap

. . . . .Bayesian Tests

. . . . .Bayesian Intervals

. . .P1

. . . . . . . .P2

. . . .P3

. . . .P4

(b) Bayes’ rule estimator with squared error loss

Bayes’ rule estimator with squared error loss is posterior mean. Note thatthe mean of Gamma(α, β) is αβ.

π(θ|x) = Gamma(α+ n − 1,

1

β−1 +∑n

i=1 xi

)E[θ|x] = E[π(θ|x)]

=α+ n − 1

β−1 +∑n

i=1 xi

Hyun Min Kang Biostatistics 602 - Lecture 25 April 18th, 2013 18 / 34

Page 70: Biostatistics 602 - Statistical Inference Lecture 25 ... · Bayesian Tests • Hypothesis testing problems can be formulated in a Bayesian model • Bayesian model includes • Sampling

..........

.....

......

.....

.....

.....

......

.....

.....

.....

......

.....

.....

.....

......

.....

......

.....

.....

.

. . . .Recap

. . . . .Bayesian Tests

. . . . .Bayesian Intervals

. . .P1

. . . . . . . .P2

. . . .P3

. . . .P4

(b) Bayes’ rule estimator with squared error loss

Bayes’ rule estimator with squared error loss is posterior mean. Note thatthe mean of Gamma(α, β) is αβ.

π(θ|x) = Gamma(α+ n − 1,

1

β−1 +∑n

i=1 xi

)

E[θ|x] = E[π(θ|x)]

=α+ n − 1

β−1 +∑n

i=1 xi

Hyun Min Kang Biostatistics 602 - Lecture 25 April 18th, 2013 18 / 34

Page 71: Biostatistics 602 - Statistical Inference Lecture 25 ... · Bayesian Tests • Hypothesis testing problems can be formulated in a Bayesian model • Bayesian model includes • Sampling

..........

.....

......

.....

.....

.....

......

.....

.....

.....

......

.....

.....

.....

......

.....

......

.....

.....

.

. . . .Recap

. . . . .Bayesian Tests

. . . . .Bayesian Intervals

. . .P1

. . . . . . . .P2

. . . .P3

. . . .P4

(b) Bayes’ rule estimator with squared error loss

Bayes’ rule estimator with squared error loss is posterior mean. Note thatthe mean of Gamma(α, β) is αβ.

π(θ|x) = Gamma(α+ n − 1,

1

β−1 +∑n

i=1 xi

)E[θ|x] = E[π(θ|x)]

=α+ n − 1

β−1 +∑n

i=1 xi

Hyun Min Kang Biostatistics 602 - Lecture 25 April 18th, 2013 18 / 34

Page 72: Biostatistics 602 - Statistical Inference Lecture 25 ... · Bayesian Tests • Hypothesis testing problems can be formulated in a Bayesian model • Bayesian model includes • Sampling

..........

.....

......

.....

.....

.....

......

.....

.....

.....

......

.....

.....

.....

......

.....

......

.....

.....

.

. . . .Recap

. . . . .Bayesian Tests

. . . . .Bayesian Intervals

. . .P1

. . . . . . . .P2

. . . .P3

. . . .P4

Practice Problem 2

.Problem..

......

Suppose X1, · · · ,Xn are iid random samples from Gamma distributionwith parameter (3, θ), which has the pdf

f(x|θ) =1

2θ3x2e−x/θ (x > 0)

You may use the result that 2∑n

i=1 Xi/θ ∼ χ26n.

(a) Derive the asymptotic size α LRT for testing H0 : θ = θ0 vs.H1 : θ = θ0.

(b) Derive the UMP level α test for H0 : θ = θ0 vs. H1 : θ = θ1, whereθ1 > θ0.

(c) Derive the UMP level α test for H0 : θ ≤ θ0 vs. H1 : θ > θ0.

Hyun Min Kang Biostatistics 602 - Lecture 25 April 18th, 2013 19 / 34

Page 73: Biostatistics 602 - Statistical Inference Lecture 25 ... · Bayesian Tests • Hypothesis testing problems can be formulated in a Bayesian model • Bayesian model includes • Sampling

..........

.....

......

.....

.....

.....

......

.....

.....

.....

......

.....

.....

.....

......

.....

......

.....

.....

.

. . . .Recap

. . . . .Bayesian Tests

. . . . .Bayesian Intervals

. . .P1

. . . . . . . .P2

. . . .P3

. . . .P4

Practice Problem 2

.Problem..

......

Suppose X1, · · · ,Xn are iid random samples from Gamma distributionwith parameter (3, θ), which has the pdf

f(x|θ) =1

2θ3x2e−x/θ (x > 0)

You may use the result that 2∑n

i=1 Xi/θ ∼ χ26n.

(a) Derive the asymptotic size α LRT for testing H0 : θ = θ0 vs.H1 : θ = θ0.

(b) Derive the UMP level α test for H0 : θ = θ0 vs. H1 : θ = θ1, whereθ1 > θ0.

(c) Derive the UMP level α test for H0 : θ ≤ θ0 vs. H1 : θ > θ0.

Hyun Min Kang Biostatistics 602 - Lecture 25 April 18th, 2013 19 / 34

Page 74: Biostatistics 602 - Statistical Inference Lecture 25 ... · Bayesian Tests • Hypothesis testing problems can be formulated in a Bayesian model • Bayesian model includes • Sampling

..........

.....

......

.....

.....

.....

......

.....

.....

.....

......

.....

.....

.....

......

.....

......

.....

.....

.

. . . .Recap

. . . . .Bayesian Tests

. . . . .Bayesian Intervals

. . .P1

. . . . . . . .P2

. . . .P3

. . . .P4

Practice Problem 2

.Problem..

......

Suppose X1, · · · ,Xn are iid random samples from Gamma distributionwith parameter (3, θ), which has the pdf

f(x|θ) =1

2θ3x2e−x/θ (x > 0)

You may use the result that 2∑n

i=1 Xi/θ ∼ χ26n.

(a) Derive the asymptotic size α LRT for testing H0 : θ = θ0 vs.H1 : θ = θ0.

(b) Derive the UMP level α test for H0 : θ = θ0 vs. H1 : θ = θ1, whereθ1 > θ0.

(c) Derive the UMP level α test for H0 : θ ≤ θ0 vs. H1 : θ > θ0.

Hyun Min Kang Biostatistics 602 - Lecture 25 April 18th, 2013 19 / 34

Page 75: Biostatistics 602 - Statistical Inference Lecture 25 ... · Bayesian Tests • Hypothesis testing problems can be formulated in a Bayesian model • Bayesian model includes • Sampling

..........

.....

......

.....

.....

.....

......

.....

.....

.....

......

.....

.....

.....

......

.....

......

.....

.....

.

. . . .Recap

. . . . .Bayesian Tests

. . . . .Bayesian Intervals

. . .P1

. . . . . . . .P2

. . . .P3

. . . .P4

Practice Problem 2

.Problem..

......

Suppose X1, · · · ,Xn are iid random samples from Gamma distributionwith parameter (3, θ), which has the pdf

f(x|θ) =1

2θ3x2e−x/θ (x > 0)

You may use the result that 2∑n

i=1 Xi/θ ∼ χ26n.

(a) Derive the asymptotic size α LRT for testing H0 : θ = θ0 vs.H1 : θ = θ0.

(b) Derive the UMP level α test for H0 : θ = θ0 vs. H1 : θ = θ1, whereθ1 > θ0.

(c) Derive the UMP level α test for H0 : θ ≤ θ0 vs. H1 : θ > θ0.

Hyun Min Kang Biostatistics 602 - Lecture 25 April 18th, 2013 19 / 34

Page 76: Biostatistics 602 - Statistical Inference Lecture 25 ... · Bayesian Tests • Hypothesis testing problems can be formulated in a Bayesian model • Bayesian model includes • Sampling

..........

.....

......

.....

.....

.....

......

.....

.....

.....

......

.....

.....

.....

......

.....

......

.....

.....

.

. . . .Recap

. . . . .Bayesian Tests

. . . . .Bayesian Intervals

. . .P1

. . . . . . . .P2

. . . .P3

. . . .P4

Solution (a) - Obtaining MLEs

L(θ|x) =

n∏i=1

[1

2θ3x2i e−xi/θ

]

l(θ|x) =

n∑i=1

[− log 2− 3 log θ + 2 log xi −

xiθ

]= −n log 2− 3n log θ + 2

n∑i=1

log xi −1

θ

n∑i=1

xi

l′(θ|x) = −3nθ

+1

θ2

n∑i=1

xi = 0

θ =1

3n

n∑i=1

xi

Hyun Min Kang Biostatistics 602 - Lecture 25 April 18th, 2013 20 / 34

Page 77: Biostatistics 602 - Statistical Inference Lecture 25 ... · Bayesian Tests • Hypothesis testing problems can be formulated in a Bayesian model • Bayesian model includes • Sampling

..........

.....

......

.....

.....

.....

......

.....

.....

.....

......

.....

.....

.....

......

.....

......

.....

.....

.

. . . .Recap

. . . . .Bayesian Tests

. . . . .Bayesian Intervals

. . .P1

. . . . . . . .P2

. . . .P3

. . . .P4

Solution (a) - Obtaining MLEs

L(θ|x) =

n∏i=1

[1

2θ3x2i e−xi/θ

]

l(θ|x) =

n∑i=1

[− log 2− 3 log θ + 2 log xi −

xiθ

]

= −n log 2− 3n log θ + 2

n∑i=1

log xi −1

θ

n∑i=1

xi

l′(θ|x) = −3nθ

+1

θ2

n∑i=1

xi = 0

θ =1

3n

n∑i=1

xi

Hyun Min Kang Biostatistics 602 - Lecture 25 April 18th, 2013 20 / 34

Page 78: Biostatistics 602 - Statistical Inference Lecture 25 ... · Bayesian Tests • Hypothesis testing problems can be formulated in a Bayesian model • Bayesian model includes • Sampling

..........

.....

......

.....

.....

.....

......

.....

.....

.....

......

.....

.....

.....

......

.....

......

.....

.....

.

. . . .Recap

. . . . .Bayesian Tests

. . . . .Bayesian Intervals

. . .P1

. . . . . . . .P2

. . . .P3

. . . .P4

Solution (a) - Obtaining MLEs

L(θ|x) =

n∏i=1

[1

2θ3x2i e−xi/θ

]

l(θ|x) =

n∑i=1

[− log 2− 3 log θ + 2 log xi −

xiθ

]= −n log 2− 3n log θ + 2

n∑i=1

log xi −1

θ

n∑i=1

xi

l′(θ|x) = −3nθ

+1

θ2

n∑i=1

xi = 0

θ =1

3n

n∑i=1

xi

Hyun Min Kang Biostatistics 602 - Lecture 25 April 18th, 2013 20 / 34

Page 79: Biostatistics 602 - Statistical Inference Lecture 25 ... · Bayesian Tests • Hypothesis testing problems can be formulated in a Bayesian model • Bayesian model includes • Sampling

..........

.....

......

.....

.....

.....

......

.....

.....

.....

......

.....

.....

.....

......

.....

......

.....

.....

.

. . . .Recap

. . . . .Bayesian Tests

. . . . .Bayesian Intervals

. . .P1

. . . . . . . .P2

. . . .P3

. . . .P4

Solution (a) - Obtaining MLEs

L(θ|x) =

n∏i=1

[1

2θ3x2i e−xi/θ

]

l(θ|x) =

n∑i=1

[− log 2− 3 log θ + 2 log xi −

xiθ

]= −n log 2− 3n log θ + 2

n∑i=1

log xi −1

θ

n∑i=1

xi

l′(θ|x) = −3nθ

+1

θ2

n∑i=1

xi = 0

θ =1

3n

n∑i=1

xi

Hyun Min Kang Biostatistics 602 - Lecture 25 April 18th, 2013 20 / 34

Page 80: Biostatistics 602 - Statistical Inference Lecture 25 ... · Bayesian Tests • Hypothesis testing problems can be formulated in a Bayesian model • Bayesian model includes • Sampling

..........

.....

......

.....

.....

.....

......

.....

.....

.....

......

.....

.....

.....

......

.....

......

.....

.....

.

. . . .Recap

. . . . .Bayesian Tests

. . . . .Bayesian Intervals

. . .P1

. . . . . . . .P2

. . . .P3

. . . .P4

Solution (a) - Obtaining MLEs

L(θ|x) =

n∏i=1

[1

2θ3x2i e−xi/θ

]

l(θ|x) =

n∑i=1

[− log 2− 3 log θ + 2 log xi −

xiθ

]= −n log 2− 3n log θ + 2

n∑i=1

log xi −1

θ

n∑i=1

xi

l′(θ|x) = −3nθ

+1

θ2

n∑i=1

xi = 0

θ =1

3n

n∑i=1

xi

Hyun Min Kang Biostatistics 602 - Lecture 25 April 18th, 2013 20 / 34

Page 81: Biostatistics 602 - Statistical Inference Lecture 25 ... · Bayesian Tests • Hypothesis testing problems can be formulated in a Bayesian model • Bayesian model includes • Sampling

..........

.....

......

.....

.....

.....

......

.....

.....

.....

......

.....

.....

.....

......

.....

......

.....

.....

.

. . . .Recap

. . . . .Bayesian Tests

. . . . .Bayesian Intervals

. . .P1

. . . . . . . .P2

. . . .P3

. . . .P4

Solution (a) - Obtaining MLEs

l′′(θ|x)∣∣θ=θ

=3nθ2

− 2

θ3

n∑i=1

xi

∣∣∣∣∣θ=θ

=3nθ2

− 6nθ2

< 0

Because L(θ|x) → 0 as θ approaches zero or infinity, θ = 13n∑n

i=1 xi.

Hyun Min Kang Biostatistics 602 - Lecture 25 April 18th, 2013 21 / 34

Page 82: Biostatistics 602 - Statistical Inference Lecture 25 ... · Bayesian Tests • Hypothesis testing problems can be formulated in a Bayesian model • Bayesian model includes • Sampling

..........

.....

......

.....

.....

.....

......

.....

.....

.....

......

.....

.....

.....

......

.....

......

.....

.....

.

. . . .Recap

. . . . .Bayesian Tests

. . . . .Bayesian Intervals

. . .P1

. . . . . . . .P2

. . . .P3

. . . .P4

Solution (a) - Obtaining MLEs

l′′(θ|x)∣∣θ=θ

=3nθ2

− 2

θ3

n∑i=1

xi

∣∣∣∣∣θ=θ

=3nθ2

− 6nθ2

< 0

Because L(θ|x) → 0 as θ approaches zero or infinity, θ = 13n∑n

i=1 xi.

Hyun Min Kang Biostatistics 602 - Lecture 25 April 18th, 2013 21 / 34

Page 83: Biostatistics 602 - Statistical Inference Lecture 25 ... · Bayesian Tests • Hypothesis testing problems can be formulated in a Bayesian model • Bayesian model includes • Sampling

..........

.....

......

.....

.....

.....

......

.....

.....

.....

......

.....

.....

.....

......

.....

......

.....

.....

.

. . . .Recap

. . . . .Bayesian Tests

. . . . .Bayesian Intervals

. . .P1

. . . . . . . .P2

. . . .P3

. . . .P4

Solution (a) - Obtaining MLEs

l′′(θ|x)∣∣θ=θ

=3nθ2

− 2

θ3

n∑i=1

xi

∣∣∣∣∣θ=θ

=3nθ2

− 6nθ2

< 0

Because L(θ|x) → 0 as θ approaches zero or infinity, θ = 13n∑n

i=1 xi.

Hyun Min Kang Biostatistics 602 - Lecture 25 April 18th, 2013 21 / 34

Page 84: Biostatistics 602 - Statistical Inference Lecture 25 ... · Bayesian Tests • Hypothesis testing problems can be formulated in a Bayesian model • Bayesian model includes • Sampling

..........

.....

......

.....

.....

.....

......

.....

.....

.....

......

.....

.....

.....

......

.....

......

.....

.....

.

. . . .Recap

. . . . .Bayesian Tests

. . . . .Bayesian Intervals

. . .P1

. . . . . . . .P2

. . . .P3

. . . .P4

Solution (a) - Constructing asymptotic size α LRT

The rejection region of asymptotic size α LRT is

−2 logλ(x) = −2[l(θ0|x)− l(θ|x)

]= 6n log θ0 +

2

θ0

∑xi − 6n log θ − 2

θ

∑xi

= 6n log θ0 +2

θ0

∑xi − 6n log

(1

3n∑

xi

)− 6n > χ2

1,α

R =

x :

2

θ0

∑xi − 6n log

∑xi > χ2

1,α + 6n[1− log(3nθ0)]

=

x :∑

xi − 3nθ0 log∑

xi >θ02χ21,α + 3nθ0[1− log(3nθ0)]

Hyun Min Kang Biostatistics 602 - Lecture 25 April 18th, 2013 22 / 34

Page 85: Biostatistics 602 - Statistical Inference Lecture 25 ... · Bayesian Tests • Hypothesis testing problems can be formulated in a Bayesian model • Bayesian model includes • Sampling

..........

.....

......

.....

.....

.....

......

.....

.....

.....

......

.....

.....

.....

......

.....

......

.....

.....

.

. . . .Recap

. . . . .Bayesian Tests

. . . . .Bayesian Intervals

. . .P1

. . . . . . . .P2

. . . .P3

. . . .P4

Solution (a) - Constructing asymptotic size α LRT

The rejection region of asymptotic size α LRT is

−2 logλ(x) = −2[l(θ0|x)− l(θ|x)

]

= 6n log θ0 +2

θ0

∑xi − 6n log θ − 2

θ

∑xi

= 6n log θ0 +2

θ0

∑xi − 6n log

(1

3n∑

xi

)− 6n > χ2

1,α

R =

x :

2

θ0

∑xi − 6n log

∑xi > χ2

1,α + 6n[1− log(3nθ0)]

=

x :∑

xi − 3nθ0 log∑

xi >θ02χ21,α + 3nθ0[1− log(3nθ0)]

Hyun Min Kang Biostatistics 602 - Lecture 25 April 18th, 2013 22 / 34

Page 86: Biostatistics 602 - Statistical Inference Lecture 25 ... · Bayesian Tests • Hypothesis testing problems can be formulated in a Bayesian model • Bayesian model includes • Sampling

..........

.....

......

.....

.....

.....

......

.....

.....

.....

......

.....

.....

.....

......

.....

......

.....

.....

.

. . . .Recap

. . . . .Bayesian Tests

. . . . .Bayesian Intervals

. . .P1

. . . . . . . .P2

. . . .P3

. . . .P4

Solution (a) - Constructing asymptotic size α LRT

The rejection region of asymptotic size α LRT is

−2 logλ(x) = −2[l(θ0|x)− l(θ|x)

]= 6n log θ0 +

2

θ0

∑xi − 6n log θ − 2

θ

∑xi

= 6n log θ0 +2

θ0

∑xi − 6n log

(1

3n∑

xi

)− 6n > χ2

1,α

R =

x :

2

θ0

∑xi − 6n log

∑xi > χ2

1,α + 6n[1− log(3nθ0)]

=

x :∑

xi − 3nθ0 log∑

xi >θ02χ21,α + 3nθ0[1− log(3nθ0)]

Hyun Min Kang Biostatistics 602 - Lecture 25 April 18th, 2013 22 / 34

Page 87: Biostatistics 602 - Statistical Inference Lecture 25 ... · Bayesian Tests • Hypothesis testing problems can be formulated in a Bayesian model • Bayesian model includes • Sampling

..........

.....

......

.....

.....

.....

......

.....

.....

.....

......

.....

.....

.....

......

.....

......

.....

.....

.

. . . .Recap

. . . . .Bayesian Tests

. . . . .Bayesian Intervals

. . .P1

. . . . . . . .P2

. . . .P3

. . . .P4

Solution (a) - Constructing asymptotic size α LRT

The rejection region of asymptotic size α LRT is

−2 logλ(x) = −2[l(θ0|x)− l(θ|x)

]= 6n log θ0 +

2

θ0

∑xi − 6n log θ − 2

θ

∑xi

= 6n log θ0 +2

θ0

∑xi − 6n log

(1

3n∑

xi

)− 6n > χ2

1,α

R =

x :

2

θ0

∑xi − 6n log

∑xi > χ2

1,α + 6n[1− log(3nθ0)]

=

x :∑

xi − 3nθ0 log∑

xi >θ02χ21,α + 3nθ0[1− log(3nθ0)]

Hyun Min Kang Biostatistics 602 - Lecture 25 April 18th, 2013 22 / 34

Page 88: Biostatistics 602 - Statistical Inference Lecture 25 ... · Bayesian Tests • Hypothesis testing problems can be formulated in a Bayesian model • Bayesian model includes • Sampling

..........

.....

......

.....

.....

.....

......

.....

.....

.....

......

.....

.....

.....

......

.....

......

.....

.....

.

. . . .Recap

. . . . .Bayesian Tests

. . . . .Bayesian Intervals

. . .P1

. . . . . . . .P2

. . . .P3

. . . .P4

Solution (a) - Constructing asymptotic size α LRT

The rejection region of asymptotic size α LRT is

−2 logλ(x) = −2[l(θ0|x)− l(θ|x)

]= 6n log θ0 +

2

θ0

∑xi − 6n log θ − 2

θ

∑xi

= 6n log θ0 +2

θ0

∑xi − 6n log

(1

3n∑

xi

)− 6n > χ2

1,α

R =

x :

2

θ0

∑xi − 6n log

∑xi > χ2

1,α + 6n[1− log(3nθ0)]

=

x :∑

xi − 3nθ0 log∑

xi >θ02χ21,α + 3nθ0[1− log(3nθ0)]

Hyun Min Kang Biostatistics 602 - Lecture 25 April 18th, 2013 22 / 34

Page 89: Biostatistics 602 - Statistical Inference Lecture 25 ... · Bayesian Tests • Hypothesis testing problems can be formulated in a Bayesian model • Bayesian model includes • Sampling

..........

.....

......

.....

.....

.....

......

.....

.....

.....

......

.....

.....

.....

......

.....

......

.....

.....

.

. . . .Recap

. . . . .Bayesian Tests

. . . . .Bayesian Intervals

. . .P1

. . . . . . . .P2

. . . .P3

. . . .P4

Solution (a) - Constructing asymptotic size α LRT

The rejection region of asymptotic size α LRT is

−2 logλ(x) = −2[l(θ0|x)− l(θ|x)

]= 6n log θ0 +

2

θ0

∑xi − 6n log θ − 2

θ

∑xi

= 6n log θ0 +2

θ0

∑xi − 6n log

(1

3n∑

xi

)− 6n > χ2

1,α

R =

x :

2

θ0

∑xi − 6n log

∑xi > χ2

1,α + 6n[1− log(3nθ0)]

=

x :∑

xi − 3nθ0 log∑

xi >θ02χ21,α + 3nθ0[1− log(3nθ0)]

Hyun Min Kang Biostatistics 602 - Lecture 25 April 18th, 2013 22 / 34

Page 90: Biostatistics 602 - Statistical Inference Lecture 25 ... · Bayesian Tests • Hypothesis testing problems can be formulated in a Bayesian model • Bayesian model includes • Sampling

..........

.....

......

.....

.....

.....

......

.....

.....

.....

......

.....

.....

.....

......

.....

......

.....

.....

.

. . . .Recap

. . . . .Bayesian Tests

. . . . .Bayesian Intervals

. . .P1

. . . . . . . .P2

. . . .P3

. . . .P4

Solution (b) - UMP level α test for simple hypothesis

For H0 : θ = θ0 vs. H1 : θ = θ1,

L(θ1|x)L(θ0|x)

=

12nθ3n

1exp

[−

∑xi

θ1

]∏x2i

12nθ3n

0exp

[−

∑xi

θ0

]∏x2i

=

(θ0θ1

)3nexp

[θ1 − θ0θ0θ1

∑xi

]

Hyun Min Kang Biostatistics 602 - Lecture 25 April 18th, 2013 23 / 34

Page 91: Biostatistics 602 - Statistical Inference Lecture 25 ... · Bayesian Tests • Hypothesis testing problems can be formulated in a Bayesian model • Bayesian model includes • Sampling

..........

.....

......

.....

.....

.....

......

.....

.....

.....

......

.....

.....

.....

......

.....

......

.....

.....

.

. . . .Recap

. . . . .Bayesian Tests

. . . . .Bayesian Intervals

. . .P1

. . . . . . . .P2

. . . .P3

. . . .P4

Solution (b) - UMP level α test for simple hypothesis

For H0 : θ = θ0 vs. H1 : θ = θ1,

L(θ1|x)L(θ0|x)

=

12nθ3n

1exp

[−

∑xi

θ1

]∏x2i

12nθ3n

0exp

[−

∑xi

θ0

]∏x2i

=

(θ0θ1

)3nexp

[θ1 − θ0θ0θ1

∑xi

]

Hyun Min Kang Biostatistics 602 - Lecture 25 April 18th, 2013 23 / 34

Page 92: Biostatistics 602 - Statistical Inference Lecture 25 ... · Bayesian Tests • Hypothesis testing problems can be formulated in a Bayesian model • Bayesian model includes • Sampling

..........

.....

......

.....

.....

.....

......

.....

.....

.....

......

.....

.....

.....

......

.....

......

.....

.....

.

. . . .Recap

. . . . .Bayesian Tests

. . . . .Bayesian Intervals

. . .P1

. . . . . . . .P2

. . . .P3

. . . .P4

Solution (b) - UMP level α test for simple hypothesis

For H0 : θ = θ0 vs. H1 : θ = θ1,

L(θ1|x)L(θ0|x)

=

12nθ3n

1exp

[−

∑xi

θ1

]∏x2i

12nθ3n

0exp

[−

∑xi

θ0

]∏x2i

=

(θ0θ1

)3nexp

[θ1 − θ0θ0θ1

∑xi

]

Hyun Min Kang Biostatistics 602 - Lecture 25 April 18th, 2013 23 / 34

Page 93: Biostatistics 602 - Statistical Inference Lecture 25 ... · Bayesian Tests • Hypothesis testing problems can be formulated in a Bayesian model • Bayesian model includes • Sampling

..........

.....

......

.....

.....

.....

......

.....

.....

.....

......

.....

.....

.....

......

.....

......

.....

.....

.

. . . .Recap

. . . . .Bayesian Tests

. . . . .Bayesian Intervals

. . .P1

. . . . . . . .P2

. . . .P3

. . . .P4

Solution (b) - UMP level α test (cont’d)

Let T =∑

Xi. Then under H0, 2θ0

T ∼ χ26n.

α = Pr[(

θ0θ1

)3nexp

[θ1 − θ0θ0θ1

T]> k]

= Pr(T > k∗)

So, the rejection region is

R =

x : T(x) =

∑xi >

θ02χ26n,α

Hyun Min Kang Biostatistics 602 - Lecture 25 April 18th, 2013 24 / 34

Page 94: Biostatistics 602 - Statistical Inference Lecture 25 ... · Bayesian Tests • Hypothesis testing problems can be formulated in a Bayesian model • Bayesian model includes • Sampling

..........

.....

......

.....

.....

.....

......

.....

.....

.....

......

.....

.....

.....

......

.....

......

.....

.....

.

. . . .Recap

. . . . .Bayesian Tests

. . . . .Bayesian Intervals

. . .P1

. . . . . . . .P2

. . . .P3

. . . .P4

Solution (b) - UMP level α test (cont’d)

Let T =∑

Xi. Then under H0, 2θ0

T ∼ χ26n.

α = Pr[(

θ0θ1

)3nexp

[θ1 − θ0θ0θ1

T]> k]

= Pr(T > k∗)

So, the rejection region is

R =

x : T(x) =

∑xi >

θ02χ26n,α

Hyun Min Kang Biostatistics 602 - Lecture 25 April 18th, 2013 24 / 34

Page 95: Biostatistics 602 - Statistical Inference Lecture 25 ... · Bayesian Tests • Hypothesis testing problems can be formulated in a Bayesian model • Bayesian model includes • Sampling

..........

.....

......

.....

.....

.....

......

.....

.....

.....

......

.....

.....

.....

......

.....

......

.....

.....

.

. . . .Recap

. . . . .Bayesian Tests

. . . . .Bayesian Intervals

. . .P1

. . . . . . . .P2

. . . .P3

. . . .P4

Solution (b) - UMP level α test (cont’d)

Let T =∑

Xi. Then under H0, 2θ0

T ∼ χ26n.

α = Pr[(

θ0θ1

)3nexp

[θ1 − θ0θ0θ1

T]> k]

= Pr(T > k∗)

So, the rejection region is

R =

x : T(x) =

∑xi >

θ02χ26n,α

Hyun Min Kang Biostatistics 602 - Lecture 25 April 18th, 2013 24 / 34

Page 96: Biostatistics 602 - Statistical Inference Lecture 25 ... · Bayesian Tests • Hypothesis testing problems can be formulated in a Bayesian model • Bayesian model includes • Sampling

..........

.....

......

.....

.....

.....

......

.....

.....

.....

......

.....

.....

.....

......

.....

......

.....

.....

.

. . . .Recap

. . . . .Bayesian Tests

. . . . .Bayesian Intervals

. . .P1

. . . . . . . .P2

. . . .P3

. . . .P4

Solution (c) - UMP level α test for composite hypothesis

We need to check whether T has MLR. Because Y = 2T/θ ∼ χ26n.

fY(y|θ) =1

23nΓ(3n)y3n−1e−y/2

fT(t|θ) =1

23n−1Γ(3n)θ

(2tθ

)3n−1

e−t/θ =1

Γ(3n)θ

(tθ

)3n−1

e−t/θ

For arbitrary θ1 < θ2,

fT(t|θ2)fT(t|θ1)

=

1Γ(3n)θ2

(tθ2

)3n−1e−t/θ2

1Γ(3n)θ1

(tθ1

)3n−1e−t/θ1

=

(θ1θ2

)3nexp

[θ2 − θ1θ1θ2

t]

is an increasing function of t. This T has MLR property.

Hyun Min Kang Biostatistics 602 - Lecture 25 April 18th, 2013 25 / 34

Page 97: Biostatistics 602 - Statistical Inference Lecture 25 ... · Bayesian Tests • Hypothesis testing problems can be formulated in a Bayesian model • Bayesian model includes • Sampling

..........

.....

......

.....

.....

.....

......

.....

.....

.....

......

.....

.....

.....

......

.....

......

.....

.....

.

. . . .Recap

. . . . .Bayesian Tests

. . . . .Bayesian Intervals

. . .P1

. . . . . . . .P2

. . . .P3

. . . .P4

Solution (c) - UMP level α test for composite hypothesis

We need to check whether T has MLR. Because Y = 2T/θ ∼ χ26n.

fY(y|θ) =1

23nΓ(3n)y3n−1e−y/2

fT(t|θ) =1

23n−1Γ(3n)θ

(2tθ

)3n−1

e−t/θ =1

Γ(3n)θ

(tθ

)3n−1

e−t/θ

For arbitrary θ1 < θ2,

fT(t|θ2)fT(t|θ1)

=

1Γ(3n)θ2

(tθ2

)3n−1e−t/θ2

1Γ(3n)θ1

(tθ1

)3n−1e−t/θ1

=

(θ1θ2

)3nexp

[θ2 − θ1θ1θ2

t]

is an increasing function of t. This T has MLR property.

Hyun Min Kang Biostatistics 602 - Lecture 25 April 18th, 2013 25 / 34

Page 98: Biostatistics 602 - Statistical Inference Lecture 25 ... · Bayesian Tests • Hypothesis testing problems can be formulated in a Bayesian model • Bayesian model includes • Sampling

..........

.....

......

.....

.....

.....

......

.....

.....

.....

......

.....

.....

.....

......

.....

......

.....

.....

.

. . . .Recap

. . . . .Bayesian Tests

. . . . .Bayesian Intervals

. . .P1

. . . . . . . .P2

. . . .P3

. . . .P4

Solution (c) - UMP level α test for composite hypothesis

We need to check whether T has MLR. Because Y = 2T/θ ∼ χ26n.

fY(y|θ) =1

23nΓ(3n)y3n−1e−y/2

fT(t|θ) =1

23n−1Γ(3n)θ

(2tθ

)3n−1

e−t/θ =1

Γ(3n)θ

(tθ

)3n−1

e−t/θ

For arbitrary θ1 < θ2,

fT(t|θ2)fT(t|θ1)

=

1Γ(3n)θ2

(tθ2

)3n−1e−t/θ2

1Γ(3n)θ1

(tθ1

)3n−1e−t/θ1

=

(θ1θ2

)3nexp

[θ2 − θ1θ1θ2

t]

is an increasing function of t. This T has MLR property.

Hyun Min Kang Biostatistics 602 - Lecture 25 April 18th, 2013 25 / 34

Page 99: Biostatistics 602 - Statistical Inference Lecture 25 ... · Bayesian Tests • Hypothesis testing problems can be formulated in a Bayesian model • Bayesian model includes • Sampling

..........

.....

......

.....

.....

.....

......

.....

.....

.....

......

.....

.....

.....

......

.....

......

.....

.....

.

. . . .Recap

. . . . .Bayesian Tests

. . . . .Bayesian Intervals

. . .P1

. . . . . . . .P2

. . . .P3

. . . .P4

Solution (c) - UMP level α test for composite hypothesis

We need to check whether T has MLR. Because Y = 2T/θ ∼ χ26n.

fY(y|θ) =1

23nΓ(3n)y3n−1e−y/2

fT(t|θ) =1

23n−1Γ(3n)θ

(2tθ

)3n−1

e−t/θ =1

Γ(3n)θ

(tθ

)3n−1

e−t/θ

For arbitrary θ1 < θ2,

fT(t|θ2)fT(t|θ1)

=

1Γ(3n)θ2

(tθ2

)3n−1e−t/θ2

1Γ(3n)θ1

(tθ1

)3n−1e−t/θ1

=

(θ1θ2

)3nexp

[θ2 − θ1θ1θ2

t]

is an increasing function of t. This T has MLR property.

Hyun Min Kang Biostatistics 602 - Lecture 25 April 18th, 2013 25 / 34

Page 100: Biostatistics 602 - Statistical Inference Lecture 25 ... · Bayesian Tests • Hypothesis testing problems can be formulated in a Bayesian model • Bayesian model includes • Sampling

..........

.....

......

.....

.....

.....

......

.....

.....

.....

......

.....

.....

.....

......

.....

......

.....

.....

.

. . . .Recap

. . . . .Bayesian Tests

. . . . .Bayesian Intervals

. . .P1

. . . . . . . .P2

. . . .P3

. . . .P4

Solution (c) - Constructing UMP level α test

Because T has MLR property, UMP level α test for H0 : θ ≤ θ0 vs.H1 : θ > θ0 has a rejection region T > k, and Pr(T > k) = α.

Therefore, the UMP level α test is identical to the answer of part (b),whose rejection is

R =

x : T(x) =

∑xi >

θ02χ26n,α

Hyun Min Kang Biostatistics 602 - Lecture 25 April 18th, 2013 26 / 34

Page 101: Biostatistics 602 - Statistical Inference Lecture 25 ... · Bayesian Tests • Hypothesis testing problems can be formulated in a Bayesian model • Bayesian model includes • Sampling

..........

.....

......

.....

.....

.....

......

.....

.....

.....

......

.....

.....

.....

......

.....

......

.....

.....

.

. . . .Recap

. . . . .Bayesian Tests

. . . . .Bayesian Intervals

. . .P1

. . . . . . . .P2

. . . .P3

. . . .P4

Solution (c) - Constructing UMP level α test

Because T has MLR property, UMP level α test for H0 : θ ≤ θ0 vs.H1 : θ > θ0 has a rejection region T > k, and Pr(T > k) = α.Therefore, the UMP level α test is identical to the answer of part (b),whose rejection is

R =

x : T(x) =

∑xi >

θ02χ26n,α

Hyun Min Kang Biostatistics 602 - Lecture 25 April 18th, 2013 26 / 34

Page 102: Biostatistics 602 - Statistical Inference Lecture 25 ... · Bayesian Tests • Hypothesis testing problems can be formulated in a Bayesian model • Bayesian model includes • Sampling

..........

.....

......

.....

.....

.....

......

.....

.....

.....

......

.....

.....

.....

......

.....

......

.....

.....

.

. . . .Recap

. . . . .Bayesian Tests

. . . . .Bayesian Intervals

. . .P1

. . . . . . . .P2

. . . .P3

. . . .P4

Solution (c) - Constructing UMP level α test

Because T has MLR property, UMP level α test for H0 : θ ≤ θ0 vs.H1 : θ > θ0 has a rejection region T > k, and Pr(T > k) = α.Therefore, the UMP level α test is identical to the answer of part (b),whose rejection is

R =

x : T(x) =

∑xi >

θ02χ26n,α

Hyun Min Kang Biostatistics 602 - Lecture 25 April 18th, 2013 26 / 34

Page 103: Biostatistics 602 - Statistical Inference Lecture 25 ... · Bayesian Tests • Hypothesis testing problems can be formulated in a Bayesian model • Bayesian model includes • Sampling

..........

.....

......

.....

.....

.....

......

.....

.....

.....

......

.....

.....

.....

......

.....

......

.....

.....

.

. . . .Recap

. . . . .Bayesian Tests

. . . . .Bayesian Intervals

. . .P1

. . . . . . . .P2

. . . .P3

. . . .P4

Practice Problem 3.Problem..

......

Let (X1,Y1), · · · , (Xn,Yn) be a random samples from a bivariate normal(XiYi

)∼ N

([µXµY

],

[σ2

X ρσXσYρσXσY σ2

Y

])We are interested in testing H0 : µX = µY vs. H1 : µX = µY.

(a) Show that the random variables Wi = Xi − Yi are iid N (µW, σ2W).

(b) Show that the above hypothesis can be tested with the statistic

TW =W√S2

W/n

where W = 1n∑n

i=1 Wi and S2W = 1

n−1

∑ni=1(Wi −W)2. Furthermore,

show that, under H0, TW follows the Student’s t distribution withn − 1 degrees of freedom.

Hyun Min Kang Biostatistics 602 - Lecture 25 April 18th, 2013 27 / 34

Page 104: Biostatistics 602 - Statistical Inference Lecture 25 ... · Bayesian Tests • Hypothesis testing problems can be formulated in a Bayesian model • Bayesian model includes • Sampling

..........

.....

......

.....

.....

.....

......

.....

.....

.....

......

.....

.....

.....

......

.....

......

.....

.....

.

. . . .Recap

. . . . .Bayesian Tests

. . . . .Bayesian Intervals

. . .P1

. . . . . . . .P2

. . . .P3

. . . .P4

Practice Problem 3.Problem..

......

Let (X1,Y1), · · · , (Xn,Yn) be a random samples from a bivariate normal(XiYi

)∼ N

([µXµY

],

[σ2

X ρσXσYρσXσY σ2

Y

])We are interested in testing H0 : µX = µY vs. H1 : µX = µY.(a) Show that the random variables Wi = Xi − Yi are iid N (µW, σ2

W).

(b) Show that the above hypothesis can be tested with the statistic

TW =W√S2

W/n

where W = 1n∑n

i=1 Wi and S2W = 1

n−1

∑ni=1(Wi −W)2. Furthermore,

show that, under H0, TW follows the Student’s t distribution withn − 1 degrees of freedom.

Hyun Min Kang Biostatistics 602 - Lecture 25 April 18th, 2013 27 / 34

Page 105: Biostatistics 602 - Statistical Inference Lecture 25 ... · Bayesian Tests • Hypothesis testing problems can be formulated in a Bayesian model • Bayesian model includes • Sampling

..........

.....

......

.....

.....

.....

......

.....

.....

.....

......

.....

.....

.....

......

.....

......

.....

.....

.

. . . .Recap

. . . . .Bayesian Tests

. . . . .Bayesian Intervals

. . .P1

. . . . . . . .P2

. . . .P3

. . . .P4

Practice Problem 3.Problem..

......

Let (X1,Y1), · · · , (Xn,Yn) be a random samples from a bivariate normal(XiYi

)∼ N

([µXµY

],

[σ2

X ρσXσYρσXσY σ2

Y

])We are interested in testing H0 : µX = µY vs. H1 : µX = µY.(a) Show that the random variables Wi = Xi − Yi are iid N (µW, σ2

W).(b) Show that the above hypothesis can be tested with the statistic

TW =W√S2

W/n

where W = 1n∑n

i=1 Wi and S2W = 1

n−1

∑ni=1(Wi −W)2. Furthermore,

show that, under H0, TW follows the Student’s t distribution withn − 1 degrees of freedom.

Hyun Min Kang Biostatistics 602 - Lecture 25 April 18th, 2013 27 / 34

Page 106: Biostatistics 602 - Statistical Inference Lecture 25 ... · Bayesian Tests • Hypothesis testing problems can be formulated in a Bayesian model • Bayesian model includes • Sampling

..........

.....

......

.....

.....

.....

......

.....

.....

.....

......

.....

.....

.....

......

.....

......

.....

.....

.

. . . .Recap

. . . . .Bayesian Tests

. . . . .Bayesian Intervals

. . .P1

. . . . . . . .P2

. . . .P3

. . . .P4

Practice Problem 3.Problem..

......

Let (X1,Y1), · · · , (Xn,Yn) be a random samples from a bivariate normal(XiYi

)∼ N

([µXµY

],

[σ2

X ρσXσYρσXσY σ2

Y

])We are interested in testing H0 : µX = µY vs. H1 : µX = µY.(a) Show that the random variables Wi = Xi − Yi are iid N (µW, σ2

W).(b) Show that the above hypothesis can be tested with the statistic

TW =W√S2

W/n

where W = 1n∑n

i=1 Wi and S2W = 1

n−1

∑ni=1(Wi −W)2. Furthermore,

show that, under H0, TW follows the Student’s t distribution withn − 1 degrees of freedom.

Hyun Min Kang Biostatistics 602 - Lecture 25 April 18th, 2013 27 / 34

Page 107: Biostatistics 602 - Statistical Inference Lecture 25 ... · Bayesian Tests • Hypothesis testing problems can be formulated in a Bayesian model • Bayesian model includes • Sampling

..........

.....

......

.....

.....

.....

......

.....

.....

.....

......

.....

.....

.....

......

.....

......

.....

.....

.

. . . .Recap

. . . . .Bayesian Tests

. . . . .Bayesian Intervals

. . .P1

. . . . . . . .P2

. . . .P3

. . . .P4

Practice Problem 3.Problem..

......

Let (X1,Y1), · · · , (Xn,Yn) be a random samples from a bivariate normal(XiYi

)∼ N

([µXµY

],

[σ2

X ρσXσYρσXσY σ2

Y

])We are interested in testing H0 : µX = µY vs. H1 : µX = µY.(a) Show that the random variables Wi = Xi − Yi are iid N (µW, σ2

W).(b) Show that the above hypothesis can be tested with the statistic

TW =W√S2

W/n

where W = 1n∑n

i=1 Wi and S2W = 1

n−1

∑ni=1(Wi −W)2. Furthermore,

show that, under H0, TW follows the Student’s t distribution withn − 1 degrees of freedom.

Hyun Min Kang Biostatistics 602 - Lecture 25 April 18th, 2013 27 / 34

Page 108: Biostatistics 602 - Statistical Inference Lecture 25 ... · Bayesian Tests • Hypothesis testing problems can be formulated in a Bayesian model • Bayesian model includes • Sampling

..........

.....

......

.....

.....

.....

......

.....

.....

.....

......

.....

.....

.....

......

.....

......

.....

.....

.

. . . .Recap

. . . . .Bayesian Tests

. . . . .Bayesian Intervals

. . .P1

. . . . . . . .P2

. . . .P3

. . . .P4

Solution (a)

To solve Problem (a), we first need to know that, if Z ∼ N (m,Σ), then

AZ ∼ N (Am,AΣAT)

Let Z = [Xi Yi]T, m = [µX µY]T, and A = [1 − 1]. Then

AZ = Xi − Yi = Wi

∼ N (Am,AΣAT)

= N (µX − µY, σ2X − 2ρσXσY + σ2

Y)

= N (µW, σ2W)

Hyun Min Kang Biostatistics 602 - Lecture 25 April 18th, 2013 28 / 34

Page 109: Biostatistics 602 - Statistical Inference Lecture 25 ... · Bayesian Tests • Hypothesis testing problems can be formulated in a Bayesian model • Bayesian model includes • Sampling

..........

.....

......

.....

.....

.....

......

.....

.....

.....

......

.....

.....

.....

......

.....

......

.....

.....

.

. . . .Recap

. . . . .Bayesian Tests

. . . . .Bayesian Intervals

. . .P1

. . . . . . . .P2

. . . .P3

. . . .P4

Solution (a)

To solve Problem (a), we first need to know that, if Z ∼ N (m,Σ), then

AZ ∼ N (Am,AΣAT)

Let Z = [Xi Yi]T, m = [µX µY]T, and A = [1 − 1]. Then

AZ = Xi − Yi = Wi

∼ N (Am,AΣAT)

= N (µX − µY, σ2X − 2ρσXσY + σ2

Y)

= N (µW, σ2W)

Hyun Min Kang Biostatistics 602 - Lecture 25 April 18th, 2013 28 / 34

Page 110: Biostatistics 602 - Statistical Inference Lecture 25 ... · Bayesian Tests • Hypothesis testing problems can be formulated in a Bayesian model • Bayesian model includes • Sampling

..........

.....

......

.....

.....

.....

......

.....

.....

.....

......

.....

.....

.....

......

.....

......

.....

.....

.

. . . .Recap

. . . . .Bayesian Tests

. . . . .Bayesian Intervals

. . .P1

. . . . . . . .P2

. . . .P3

. . . .P4

Solution (a)

To solve Problem (a), we first need to know that, if Z ∼ N (m,Σ), then

AZ ∼ N (Am,AΣAT)

Let Z = [Xi Yi]T, m = [µX µY]T, and A = [1 − 1]. Then

AZ = Xi − Yi = Wi

∼ N (Am,AΣAT)

= N (µX − µY, σ2X − 2ρσXσY + σ2

Y)

= N (µW, σ2W)

Hyun Min Kang Biostatistics 602 - Lecture 25 April 18th, 2013 28 / 34

Page 111: Biostatistics 602 - Statistical Inference Lecture 25 ... · Bayesian Tests • Hypothesis testing problems can be formulated in a Bayesian model • Bayesian model includes • Sampling

..........

.....

......

.....

.....

.....

......

.....

.....

.....

......

.....

.....

.....

......

.....

......

.....

.....

.

. . . .Recap

. . . . .Bayesian Tests

. . . . .Bayesian Intervals

. . .P1

. . . . . . . .P2

. . . .P3

. . . .P4

Solution (a)

To solve Problem (a), we first need to know that, if Z ∼ N (m,Σ), then

AZ ∼ N (Am,AΣAT)

Let Z = [Xi Yi]T, m = [µX µY]T, and A = [1 − 1]. Then

AZ = Xi − Yi = Wi

∼ N (Am,AΣAT)

= N (µX − µY, σ2X − 2ρσXσY + σ2

Y)

= N (µW, σ2W)

Hyun Min Kang Biostatistics 602 - Lecture 25 April 18th, 2013 28 / 34

Page 112: Biostatistics 602 - Statistical Inference Lecture 25 ... · Bayesian Tests • Hypothesis testing problems can be formulated in a Bayesian model • Bayesian model includes • Sampling

..........

.....

......

.....

.....

.....

......

.....

.....

.....

......

.....

.....

.....

......

.....

......

.....

.....

.

. . . .Recap

. . . . .Bayesian Tests

. . . . .Bayesian Intervals

. . .P1

. . . . . . . .P2

. . . .P3

. . . .P4

Solution (a)

To solve Problem (a), we first need to know that, if Z ∼ N (m,Σ), then

AZ ∼ N (Am,AΣAT)

Let Z = [Xi Yi]T, m = [µX µY]T, and A = [1 − 1]. Then

AZ = Xi − Yi = Wi

∼ N (Am,AΣAT)

= N (µX − µY, σ2X − 2ρσXσY + σ2

Y)

= N (µW, σ2W)

Hyun Min Kang Biostatistics 602 - Lecture 25 April 18th, 2013 28 / 34

Page 113: Biostatistics 602 - Statistical Inference Lecture 25 ... · Bayesian Tests • Hypothesis testing problems can be formulated in a Bayesian model • Bayesian model includes • Sampling

..........

.....

......

.....

.....

.....

......

.....

.....

.....

......

.....

.....

.....

......

.....

......

.....

.....

.

. . . .Recap

. . . . .Bayesian Tests

. . . . .Bayesian Intervals

. . .P1

. . . . . . . .P2

. . . .P3

. . . .P4

Solution (a)

To solve Problem (a), we first need to know that, if Z ∼ N (m,Σ), then

AZ ∼ N (Am,AΣAT)

Let Z = [Xi Yi]T, m = [µX µY]T, and A = [1 − 1]. Then

AZ = Xi − Yi = Wi

∼ N (Am,AΣAT)

= N (µX − µY, σ2X − 2ρσXσY + σ2

Y)

= N (µW, σ2W)

Hyun Min Kang Biostatistics 602 - Lecture 25 April 18th, 2013 28 / 34

Page 114: Biostatistics 602 - Statistical Inference Lecture 25 ... · Bayesian Tests • Hypothesis testing problems can be formulated in a Bayesian model • Bayesian model includes • Sampling

..........

.....

......

.....

.....

.....

......

.....

.....

.....

......

.....

.....

.....

......

.....

......

.....

.....

.

. . . .Recap

. . . . .Bayesian Tests

. . . . .Bayesian Intervals

. . .P1

. . . . . . . .P2

. . . .P3

. . . .P4

Solution (b)

Because µW = µX − µY, testing

H0 : µX = µY vs. H1 : µX = µY

is equivalent to testing

H0 : µW = 0 vs. H1 : µW = 0

When Ui ∼ N (µ, σ2) and both mean and variance are unknown, we knowthat LRT testing H0 : µ = µ0 vs. H0 : µ = µ0 follows that

TU =U − µ0√

S2U/n

and TU follows Tn−1 under H0.

Hyun Min Kang Biostatistics 602 - Lecture 25 April 18th, 2013 29 / 34

Page 115: Biostatistics 602 - Statistical Inference Lecture 25 ... · Bayesian Tests • Hypothesis testing problems can be formulated in a Bayesian model • Bayesian model includes • Sampling

..........

.....

......

.....

.....

.....

......

.....

.....

.....

......

.....

.....

.....

......

.....

......

.....

.....

.

. . . .Recap

. . . . .Bayesian Tests

. . . . .Bayesian Intervals

. . .P1

. . . . . . . .P2

. . . .P3

. . . .P4

Solution (b)

Because µW = µX − µY, testing

H0 : µX = µY vs. H1 : µX = µY

is equivalent to testing

H0 : µW = 0 vs. H1 : µW = 0

When Ui ∼ N (µ, σ2) and both mean and variance are unknown, we knowthat LRT testing H0 : µ = µ0 vs. H0 : µ = µ0 follows that

TU =U − µ0√

S2U/n

and TU follows Tn−1 under H0.

Hyun Min Kang Biostatistics 602 - Lecture 25 April 18th, 2013 29 / 34

Page 116: Biostatistics 602 - Statistical Inference Lecture 25 ... · Bayesian Tests • Hypothesis testing problems can be formulated in a Bayesian model • Bayesian model includes • Sampling

..........

.....

......

.....

.....

.....

......

.....

.....

.....

......

.....

.....

.....

......

.....

......

.....

.....

.

. . . .Recap

. . . . .Bayesian Tests

. . . . .Bayesian Intervals

. . .P1

. . . . . . . .P2

. . . .P3

. . . .P4

Solution (b)

Because µW = µX − µY, testing

H0 : µX = µY vs. H1 : µX = µY

is equivalent to testing

H0 : µW = 0 vs. H1 : µW = 0

When Ui ∼ N (µ, σ2) and both mean and variance are unknown, we knowthat LRT testing H0 : µ = µ0 vs. H0 : µ = µ0 follows that

TU =U − µ0√

S2U/n

and TU follows Tn−1 under H0.

Hyun Min Kang Biostatistics 602 - Lecture 25 April 18th, 2013 29 / 34

Page 117: Biostatistics 602 - Statistical Inference Lecture 25 ... · Bayesian Tests • Hypothesis testing problems can be formulated in a Bayesian model • Bayesian model includes • Sampling

..........

.....

......

.....

.....

.....

......

.....

.....

.....

......

.....

.....

.....

......

.....

......

.....

.....

.

. . . .Recap

. . . . .Bayesian Tests

. . . . .Bayesian Intervals

. . .P1

. . . . . . . .P2

. . . .P3

. . . .P4

Solution (b) (cont’d)

Therefore, the LRT test for the original test, H0 : µW = 0 vs. H1 : µW = 0is

TW =W√S2

W/n

and TW follows Tn−1 under H0.

Hyun Min Kang Biostatistics 602 - Lecture 25 April 18th, 2013 30 / 34

Page 118: Biostatistics 602 - Statistical Inference Lecture 25 ... · Bayesian Tests • Hypothesis testing problems can be formulated in a Bayesian model • Bayesian model includes • Sampling

..........

.....

......

.....

.....

.....

......

.....

.....

.....

......

.....

.....

.....

......

.....

......

.....

.....

.

. . . .Recap

. . . . .Bayesian Tests

. . . . .Bayesian Intervals

. . .P1

. . . . . . . .P2

. . . .P3

. . . .P4

Practice Problem 4

.Problem..

......

Let f(x|θ) be the logistic location pdf

f(x|θ) =e(x−θ)

(1 + e(x−θ))2−∞ < x < ∞, −∞ < θ < ∞

(a) Show that this family has an MLR(b) Based on one observation X, find the most powerful size α test of

H0 : θ = 0 versus H1 : θ = 1.(c) Show that the test in part (b) is UMP size α for testing H0 : θ ≤ 0 vs.

H1 : θ > 0.

Hyun Min Kang Biostatistics 602 - Lecture 25 April 18th, 2013 31 / 34

Page 119: Biostatistics 602 - Statistical Inference Lecture 25 ... · Bayesian Tests • Hypothesis testing problems can be formulated in a Bayesian model • Bayesian model includes • Sampling

..........

.....

......

.....

.....

.....

......

.....

.....

.....

......

.....

.....

.....

......

.....

......

.....

.....

.

. . . .Recap

. . . . .Bayesian Tests

. . . . .Bayesian Intervals

. . .P1

. . . . . . . .P2

. . . .P3

. . . .P4

Practice Problem 4

.Problem..

......

Let f(x|θ) be the logistic location pdf

f(x|θ) =e(x−θ)

(1 + e(x−θ))2−∞ < x < ∞, −∞ < θ < ∞

(a) Show that this family has an MLR

(b) Based on one observation X, find the most powerful size α test ofH0 : θ = 0 versus H1 : θ = 1.

(c) Show that the test in part (b) is UMP size α for testing H0 : θ ≤ 0 vs.H1 : θ > 0.

Hyun Min Kang Biostatistics 602 - Lecture 25 April 18th, 2013 31 / 34

Page 120: Biostatistics 602 - Statistical Inference Lecture 25 ... · Bayesian Tests • Hypothesis testing problems can be formulated in a Bayesian model • Bayesian model includes • Sampling

..........

.....

......

.....

.....

.....

......

.....

.....

.....

......

.....

.....

.....

......

.....

......

.....

.....

.

. . . .Recap

. . . . .Bayesian Tests

. . . . .Bayesian Intervals

. . .P1

. . . . . . . .P2

. . . .P3

. . . .P4

Practice Problem 4

.Problem..

......

Let f(x|θ) be the logistic location pdf

f(x|θ) =e(x−θ)

(1 + e(x−θ))2−∞ < x < ∞, −∞ < θ < ∞

(a) Show that this family has an MLR(b) Based on one observation X, find the most powerful size α test of

H0 : θ = 0 versus H1 : θ = 1.

(c) Show that the test in part (b) is UMP size α for testing H0 : θ ≤ 0 vs.H1 : θ > 0.

Hyun Min Kang Biostatistics 602 - Lecture 25 April 18th, 2013 31 / 34

Page 121: Biostatistics 602 - Statistical Inference Lecture 25 ... · Bayesian Tests • Hypothesis testing problems can be formulated in a Bayesian model • Bayesian model includes • Sampling

..........

.....

......

.....

.....

.....

......

.....

.....

.....

......

.....

.....

.....

......

.....

......

.....

.....

.

. . . .Recap

. . . . .Bayesian Tests

. . . . .Bayesian Intervals

. . .P1

. . . . . . . .P2

. . . .P3

. . . .P4

Practice Problem 4

.Problem..

......

Let f(x|θ) be the logistic location pdf

f(x|θ) =e(x−θ)

(1 + e(x−θ))2−∞ < x < ∞, −∞ < θ < ∞

(a) Show that this family has an MLR(b) Based on one observation X, find the most powerful size α test of

H0 : θ = 0 versus H1 : θ = 1.(c) Show that the test in part (b) is UMP size α for testing H0 : θ ≤ 0 vs.

H1 : θ > 0.

Hyun Min Kang Biostatistics 602 - Lecture 25 April 18th, 2013 31 / 34

Page 122: Biostatistics 602 - Statistical Inference Lecture 25 ... · Bayesian Tests • Hypothesis testing problems can be formulated in a Bayesian model • Bayesian model includes • Sampling

..........

.....

......

.....

.....

.....

......

.....

.....

.....

......

.....

.....

.....

......

.....

......

.....

.....

.

. . . .Recap

. . . . .Bayesian Tests

. . . . .Bayesian Intervals

. . .P1

. . . . . . . .P2

. . . .P3

. . . .P4

Solution for (a)For θ1 < θ2,

f(x|θ2)f(x|θ1)

=

e(x−θ2)

(1+e(x−θ2))2

e(x−θ1)

(1+e(x−θ1))2

= e(θ1−θ2)

(1 + e(x−θ1)

1 + e(x−θ2)

)2

Let r(x) = (1 + ex−θ1)/(1 + ex−θ2)

r′(x) =e(x−θ1)(1 + e(x−θ2))− (1 + e(x−θ1))e(x−θ2)

(1 + e(x−θ2))2

=e(x−θ1) − e(x−θ2)

(1 + e(x−θ2))2> 0 (∵ x − θ1 > x − θ2)

Therefore, the family of X has an MLR.

Hyun Min Kang Biostatistics 602 - Lecture 25 April 18th, 2013 32 / 34

Page 123: Biostatistics 602 - Statistical Inference Lecture 25 ... · Bayesian Tests • Hypothesis testing problems can be formulated in a Bayesian model • Bayesian model includes • Sampling

..........

.....

......

.....

.....

.....

......

.....

.....

.....

......

.....

.....

.....

......

.....

......

.....

.....

.

. . . .Recap

. . . . .Bayesian Tests

. . . . .Bayesian Intervals

. . .P1

. . . . . . . .P2

. . . .P3

. . . .P4

Solution for (a)For θ1 < θ2,

f(x|θ2)f(x|θ1)

=

e(x−θ2)

(1+e(x−θ2))2

e(x−θ1)

(1+e(x−θ1))2

= e(θ1−θ2)

(1 + e(x−θ1)

1 + e(x−θ2)

)2

Let r(x) = (1 + ex−θ1)/(1 + ex−θ2)

r′(x) =e(x−θ1)(1 + e(x−θ2))− (1 + e(x−θ1))e(x−θ2)

(1 + e(x−θ2))2

=e(x−θ1) − e(x−θ2)

(1 + e(x−θ2))2> 0 (∵ x − θ1 > x − θ2)

Therefore, the family of X has an MLR.

Hyun Min Kang Biostatistics 602 - Lecture 25 April 18th, 2013 32 / 34

Page 124: Biostatistics 602 - Statistical Inference Lecture 25 ... · Bayesian Tests • Hypothesis testing problems can be formulated in a Bayesian model • Bayesian model includes • Sampling

..........

.....

......

.....

.....

.....

......

.....

.....

.....

......

.....

.....

.....

......

.....

......

.....

.....

.

. . . .Recap

. . . . .Bayesian Tests

. . . . .Bayesian Intervals

. . .P1

. . . . . . . .P2

. . . .P3

. . . .P4

Solution for (a)For θ1 < θ2,

f(x|θ2)f(x|θ1)

=

e(x−θ2)

(1+e(x−θ2))2

e(x−θ1)

(1+e(x−θ1))2

= e(θ1−θ2)

(1 + e(x−θ1)

1 + e(x−θ2)

)2

Let r(x) = (1 + ex−θ1)/(1 + ex−θ2)

r′(x) =e(x−θ1)(1 + e(x−θ2))− (1 + e(x−θ1))e(x−θ2)

(1 + e(x−θ2))2

=e(x−θ1) − e(x−θ2)

(1 + e(x−θ2))2> 0 (∵ x − θ1 > x − θ2)

Therefore, the family of X has an MLR.

Hyun Min Kang Biostatistics 602 - Lecture 25 April 18th, 2013 32 / 34

Page 125: Biostatistics 602 - Statistical Inference Lecture 25 ... · Bayesian Tests • Hypothesis testing problems can be formulated in a Bayesian model • Bayesian model includes • Sampling

..........

.....

......

.....

.....

.....

......

.....

.....

.....

......

.....

.....

.....

......

.....

......

.....

.....

.

. . . .Recap

. . . . .Bayesian Tests

. . . . .Bayesian Intervals

. . .P1

. . . . . . . .P2

. . . .P3

. . . .P4

Solution for (a)For θ1 < θ2,

f(x|θ2)f(x|θ1)

=

e(x−θ2)

(1+e(x−θ2))2

e(x−θ1)

(1+e(x−θ1))2

= e(θ1−θ2)

(1 + e(x−θ1)

1 + e(x−θ2)

)2

Let r(x) = (1 + ex−θ1)/(1 + ex−θ2)

r′(x) =e(x−θ1)(1 + e(x−θ2))− (1 + e(x−θ1))e(x−θ2)

(1 + e(x−θ2))2

=e(x−θ1) − e(x−θ2)

(1 + e(x−θ2))2> 0 (∵ x − θ1 > x − θ2)

Therefore, the family of X has an MLR.

Hyun Min Kang Biostatistics 602 - Lecture 25 April 18th, 2013 32 / 34

Page 126: Biostatistics 602 - Statistical Inference Lecture 25 ... · Bayesian Tests • Hypothesis testing problems can be formulated in a Bayesian model • Bayesian model includes • Sampling

..........

.....

......

.....

.....

.....

......

.....

.....

.....

......

.....

.....

.....

......

.....

......

.....

.....

.

. . . .Recap

. . . . .Bayesian Tests

. . . . .Bayesian Intervals

. . .P1

. . . . . . . .P2

. . . .P3

. . . .P4

Solution for (a)For θ1 < θ2,

f(x|θ2)f(x|θ1)

=

e(x−θ2)

(1+e(x−θ2))2

e(x−θ1)

(1+e(x−θ1))2

= e(θ1−θ2)

(1 + e(x−θ1)

1 + e(x−θ2)

)2

Let r(x) = (1 + ex−θ1)/(1 + ex−θ2)

r′(x) =e(x−θ1)(1 + e(x−θ2))− (1 + e(x−θ1))e(x−θ2)

(1 + e(x−θ2))2

=e(x−θ1) − e(x−θ2)

(1 + e(x−θ2))2> 0 (∵ x − θ1 > x − θ2)

Therefore, the family of X has an MLR.

Hyun Min Kang Biostatistics 602 - Lecture 25 April 18th, 2013 32 / 34

Page 127: Biostatistics 602 - Statistical Inference Lecture 25 ... · Bayesian Tests • Hypothesis testing problems can be formulated in a Bayesian model • Bayesian model includes • Sampling

..........

.....

......

.....

.....

.....

......

.....

.....

.....

......

.....

.....

.....

......

.....

......

.....

.....

.

. . . .Recap

. . . . .Bayesian Tests

. . . . .Bayesian Intervals

. . .P1

. . . . . . . .P2

. . . .P3

. . . .P4

Solution for (a)For θ1 < θ2,

f(x|θ2)f(x|θ1)

=

e(x−θ2)

(1+e(x−θ2))2

e(x−θ1)

(1+e(x−θ1))2

= e(θ1−θ2)

(1 + e(x−θ1)

1 + e(x−θ2)

)2

Let r(x) = (1 + ex−θ1)/(1 + ex−θ2)

r′(x) =e(x−θ1)(1 + e(x−θ2))− (1 + e(x−θ1))e(x−θ2)

(1 + e(x−θ2))2

=e(x−θ1) − e(x−θ2)

(1 + e(x−θ2))2> 0 (∵ x − θ1 > x − θ2)

Therefore, the family of X has an MLR.Hyun Min Kang Biostatistics 602 - Lecture 25 April 18th, 2013 32 / 34

Page 128: Biostatistics 602 - Statistical Inference Lecture 25 ... · Bayesian Tests • Hypothesis testing problems can be formulated in a Bayesian model • Bayesian model includes • Sampling

..........

.....

......

.....

.....

.....

......

.....

.....

.....

......

.....

.....

.....

......

.....

......

.....

.....

.

. . . .Recap

. . . . .Bayesian Tests

. . . . .Bayesian Intervals

. . .P1

. . . . . . . .P2

. . . .P3

. . . .P4

Solution for (b)The UMP test rejects H0 if and only if

f(x|1)f(x|0) = e

(1 + ex

1 + e(x−1)

)2

> k

1 + ex

1 + e(x−1)> k∗

1 + ex

e + ex > k∗

X > x0Because under H0, F(x|θ = 0) = ex

1+ex , the rejection region of UMP level αtest satisfies

1− F(x|θ = 0) =1

1 + ex0 = α

x0 = log(1− α

α

)

Hyun Min Kang Biostatistics 602 - Lecture 25 April 18th, 2013 33 / 34

Page 129: Biostatistics 602 - Statistical Inference Lecture 25 ... · Bayesian Tests • Hypothesis testing problems can be formulated in a Bayesian model • Bayesian model includes • Sampling

..........

.....

......

.....

.....

.....

......

.....

.....

.....

......

.....

.....

.....

......

.....

......

.....

.....

.

. . . .Recap

. . . . .Bayesian Tests

. . . . .Bayesian Intervals

. . .P1

. . . . . . . .P2

. . . .P3

. . . .P4

Solution for (b)The UMP test rejects H0 if and only if

f(x|1)f(x|0) = e

(1 + ex

1 + e(x−1)

)2

> k

1 + ex

1 + e(x−1)> k∗

1 + ex

e + ex > k∗

X > x0Because under H0, F(x|θ = 0) = ex

1+ex , the rejection region of UMP level αtest satisfies

1− F(x|θ = 0) =1

1 + ex0 = α

x0 = log(1− α

α

)

Hyun Min Kang Biostatistics 602 - Lecture 25 April 18th, 2013 33 / 34

Page 130: Biostatistics 602 - Statistical Inference Lecture 25 ... · Bayesian Tests • Hypothesis testing problems can be formulated in a Bayesian model • Bayesian model includes • Sampling

..........

.....

......

.....

.....

.....

......

.....

.....

.....

......

.....

.....

.....

......

.....

......

.....

.....

.

. . . .Recap

. . . . .Bayesian Tests

. . . . .Bayesian Intervals

. . .P1

. . . . . . . .P2

. . . .P3

. . . .P4

Solution for (b)The UMP test rejects H0 if and only if

f(x|1)f(x|0) = e

(1 + ex

1 + e(x−1)

)2

> k

1 + ex

1 + e(x−1)> k∗

1 + ex

e + ex > k∗

X > x0Because under H0, F(x|θ = 0) = ex

1+ex , the rejection region of UMP level αtest satisfies

1− F(x|θ = 0) =1

1 + ex0 = α

x0 = log(1− α

α

)

Hyun Min Kang Biostatistics 602 - Lecture 25 April 18th, 2013 33 / 34

Page 131: Biostatistics 602 - Statistical Inference Lecture 25 ... · Bayesian Tests • Hypothesis testing problems can be formulated in a Bayesian model • Bayesian model includes • Sampling

..........

.....

......

.....

.....

.....

......

.....

.....

.....

......

.....

.....

.....

......

.....

......

.....

.....

.

. . . .Recap

. . . . .Bayesian Tests

. . . . .Bayesian Intervals

. . .P1

. . . . . . . .P2

. . . .P3

. . . .P4

Solution for (b)The UMP test rejects H0 if and only if

f(x|1)f(x|0) = e

(1 + ex

1 + e(x−1)

)2

> k

1 + ex

1 + e(x−1)> k∗

1 + ex

e + ex > k∗

X > x0

Because under H0, F(x|θ = 0) = ex

1+ex , the rejection region of UMP level αtest satisfies

1− F(x|θ = 0) =1

1 + ex0 = α

x0 = log(1− α

α

)

Hyun Min Kang Biostatistics 602 - Lecture 25 April 18th, 2013 33 / 34

Page 132: Biostatistics 602 - Statistical Inference Lecture 25 ... · Bayesian Tests • Hypothesis testing problems can be formulated in a Bayesian model • Bayesian model includes • Sampling

..........

.....

......

.....

.....

.....

......

.....

.....

.....

......

.....

.....

.....

......

.....

......

.....

.....

.

. . . .Recap

. . . . .Bayesian Tests

. . . . .Bayesian Intervals

. . .P1

. . . . . . . .P2

. . . .P3

. . . .P4

Solution for (b)The UMP test rejects H0 if and only if

f(x|1)f(x|0) = e

(1 + ex

1 + e(x−1)

)2

> k

1 + ex

1 + e(x−1)> k∗

1 + ex

e + ex > k∗

X > x0Because under H0, F(x|θ = 0) = ex

1+ex , the rejection region of UMP level αtest satisfies

1− F(x|θ = 0) =1

1 + ex0 = α

x0 = log(1− α

α

)Hyun Min Kang Biostatistics 602 - Lecture 25 April 18th, 2013 33 / 34

Page 133: Biostatistics 602 - Statistical Inference Lecture 25 ... · Bayesian Tests • Hypothesis testing problems can be formulated in a Bayesian model • Bayesian model includes • Sampling

..........

.....

......

.....

.....

.....

......

.....

.....

.....

......

.....

.....

.....

......

.....

......

.....

.....

.

. . . .Recap

. . . . .Bayesian Tests

. . . . .Bayesian Intervals

. . .P1

. . . . . . . .P2

. . . .P3

. . . .P4

Solution for (c)

Because the family of X has an MLR, UMP size α for testing H0 : θ ≤ 0vs. H1 : θ > 0 should be a form of

X > x0Pr(X > x0|θ = 0) = α

Therefore, x0 = log(1−αα

), which is identical to the test defined in (b).

Hyun Min Kang Biostatistics 602 - Lecture 25 April 18th, 2013 34 / 34

Page 134: Biostatistics 602 - Statistical Inference Lecture 25 ... · Bayesian Tests • Hypothesis testing problems can be formulated in a Bayesian model • Bayesian model includes • Sampling

..........

.....

......

.....

.....

.....

......

.....

.....

.....

......

.....

.....

.....

......

.....

......

.....

.....

.

. . . .Recap

. . . . .Bayesian Tests

. . . . .Bayesian Intervals

. . .P1

. . . . . . . .P2

. . . .P3

. . . .P4

Solution for (c)

Because the family of X has an MLR, UMP size α for testing H0 : θ ≤ 0vs. H1 : θ > 0 should be a form of

X > x0Pr(X > x0|θ = 0) = α

Therefore, x0 = log(1−αα

), which is identical to the test defined in (b).

Hyun Min Kang Biostatistics 602 - Lecture 25 April 18th, 2013 34 / 34